Nothing Special   »   [go: up one dir, main page]

Jawaban 1

Download as docx, pdf, or txt
Download as docx, pdf, or txt
You are on page 1of 52

1.

After an initial pregnancy resulted in a spontaneous loss in the first trimester, your
patient is concerned about the possibility of this recurring. Which of the following is
the most appropriate answer regarding the risk of recurrence after one miscarriage?
a. It depends on the genetic makeup of the prior abortus.
b. It is no different than it was prior to the miscarriage.
c. It has increased to approximately 50%.
d. It does not increase regardless of number of prior miscarriages.
e. It depends on the gender of the prior abortus.

The answer is b. An initial spontaneous abortion, regardless of the karyotype or


gender of the child, does not change the risk of recurrence in a future pregnancy. The
rate is commonly quoted as 15% of all known pregnancies.

2. A 24-year-old woman presents with a history of one first-trimester spontaneous


abortion. Which of the following is the single most common specific chromosome
abnormality associated with first trimester miscarriage?
a. 45 X (Turner syndrome)
b. Trisomy 21 (Down syndrome)
c. Trisomy 18
d. Trisomy 16
e. 46 XXY (Klinefelter syndrome)

The answer is a. Chromosomal abnormalities are found in approximately 50% of


spontaneous abortions in the first trimester. Chromosome abnormalities become less
common in advancing pregnancy, and are found in approximately one-third of second
trimester losses and 5% of third trimester losses. Autosomal trisomy is the most
common group of chromosomal anomalies leading to first trimester miscarriage.
However, 45 X (Turner syndrome) is the most common single abnormality found.

3. A 29-year-old G3P0 presents to your office for preconception counseling. All of her
pregnancies were lost in the first trimester. She has no significant past medical or
surgical history. She should be counseled that without evaluation and treatment her
chance of having a live birth is which of the following?
a. < 20%
b. 20% to 35%
c. 40% to 50%
d. 70% to 85%
e. > 85%

The answer is c. Miscarriage risk rises with the number of prior spontaneous
abortions. Without treatment, the live birth rate approaches 50%. With treatment,
successful pregnancy rates of 70% to 85% are possible in a patient with a diagnosis of
habitual abortion, depending on the underlying cause. When cervical incompetence is
present and a cerclage is placed, success rates can approach 90%.
4. A 26-year-old G3P0030 has had three consecutive spontaneous abortions in the first
trimester. As part of an evaluation for this problem, which of the following tests is
most appropriate in the evaluation of this patient?
a. Hysterosalpingogram
b. Chromosomal analysis of the couple
c. Endometrial biopsy in the luteal phase
d. Postcoital test
e. Cervical length by ultrasonography

The answer is b. A major cause of spontaneous abortions in the first trimester is


chromosomal abnormalities. Parental chromosome anomalies account for 2% to 4%
of recurrent losses; therefore, karyotype evaluation of the parents is an important part
of the evaluation. The causes of losses in the second trimester are more likely to be
uterine or environmental in origin. Patients should also be screened for thyroid
function, diabetes mellitus, and collagen vascular disorders. There is also a correlation
between patients with a positive lupus anticoagulant and recurrent miscarriages. For
recurrent second-trimester losses, a hysterosalpingogram should be ordered to rule out
uterine structural abnormalities, such as bicornuate uterus, septate uterus, or
unicornuate uterus. Endometrial biopsy is performed to rule out an insufficiency of
the luteal phase or evidence of chronic endometritis. A postcoital test may be useful
during an infertility evaluation for couples who cannot conceive, but does not address
postconception losses. Measuring the cervical length by ultrasonography is helpful in
the management of patients with recurrent second-trimester losses caused by cervical
incompetence.

Questions 5 to 8

5. A 30-year-old G1P0 at 8 weeks’ gestation presents for her first prenatal visit. She has
no significant past medical or surgical history. A 29-year-old friend of hers just had a
baby with Down syndrome and she is concerned about her risk of having a baby with
the same problem. The patient reports no family history of genetic disorders or birth
defects. 5. You should tell her that she has an increased risk of having a baby with
Down syndrome in which of the following circumstances?
a. The age of the father of the baby is 40 years or older.
b. Her pregnancy was achieved by induction of ovulation and artificial insemination.
c. She has an incompetent cervix.
d. She has a luteal phase defect.
e. She has had three first-trimester spontaneous abortions.

The answer is e. The risk of aneuploidy is increased with multiple miscarriages not
attributable to other causes such as endocrine abnormalities or cervical incompetence.
Paternal age does not contribute significantly to aneuploidy until around age 55, and
most risks of paternal age are for point mutations. A 45 X karyotype results from loss
of chromosome material and does not involve increased risks for nondisjunctional
errors. Similarly, induced ovulation does not result in increased nondisjunction, and
hypermodel conceptions (triploidy) do not increase risk for future pregnancies.
6. You offer her a first trimester ultrasound looking for ultrasound markers associated
with Down syndrome. Which of the following ultrasound markers is most closely
associated with Down syndrome?
a. Choriod plexus cyst
b. Ventriculomegaly
c. Increased nuchal translucency (NT)
d. Intracardiac echogenic focus
e. Echogenic bowel

The answer is c. All of the markers listed are associated in some degree with Down
syndrome as well as other genetic abnormalities, but increased NT is most closely and
consistently associated. Increased NT is an early presenting feature of Down
syndrome. Guidelines for systemic measurement of NT are standardized. Specific
training and ongoing audits of examination quality are required for screening
programs in order to ensure the expected detection rate. The optimal time to schedule
NT measurement is between 12 and 13 weeks, but results are considered valid
between 10 4/7 and 13 6/7 weeks. This results in Down syndrome detection rates of
72% at a screen positive rate of 5%. Most centers use a thickness of >/= 3 mm to
define abnormal.

7. In order to increase the detection rate for Down syndrome in the first trimester, you
may also offer her which of the following tests in addition to the NT measurement?
a. α fetoprotein (AFP) serum screening
b. First trimester screen, which includes biochemical testing with serum markers
PAPP-A and free or total β-hCG, along with maternal age
c. Amniocentesis
d. Inhibin level serum screening
e. Fetal echocardiogram

The answer is b. Several large, multicenter trials have shown that, in the first
trimester, a combination of NT measurement, maternal age, and serum markers
(PAPP-A and free or total β-hCG) is a reliable test for Down syndrome, with a
detection rate of approximately 84%. Serum AFP is available as a screen for NTDs,
and should be ordered after 15 weeks. Inhibin level alone is not a screen for Down
syndrome, but may be part of a Quad screen. Amniocentesis cannot be offered until
the second trimester. Fetal echocardiogram is not reliable in the first trimester, may
not show cardiac defects in the first trimester, and is not considered a screening test
for Down syndrome. It may be ordered in fetuses suspected to have Down syndrome
based on abnormal diagnostic testing.

8. The patient has an abnormal first trimester screen with increased risk of Down
syndrome reported. What is the most appropriate next step?
a. Offer termination of the pregnancy.
b. Tell the patient that the baby will have Down syndrome.
c. Refer the patient to a high-risk specialist.
d. Refer the patient to genetic counseling.
e. Refer the patient to genetic counseling, and offer her diagnostic testing by CVS or a
second-trimester genetic amniocentesis.

The answer is e. The patient should be referred for genetic counseling and offered a
diagnostic test such as CVS or amniocentesis. Genetic counseling alone is not
adequate, and referral to a high-risk specialist is not indicated at this time. The patient
should not be told that the baby has Down syndrome, as the first trimester screen has
a 5% false positive rate, and requires follow-up diagnostic testing. The patient should
not be offered termination at this point, but it would be reasonable to offer termination
if diagnostic testing confirmed Down syndrome.

9. A 29-year-old Caucasian primigravida is 20 weeks pregnant with twins. Today, on her


routine ultrasound for fetal anatomy, she found out that she is carrying two boys. In
this patient’s case, which of the following statements correctly describes the zygosity
of this pregnancy?
a. The twins must be monozygotic since they are both the same gender.
b. If division of these twins occurred after formation of the embryonic disk, the twins
will be conjoined.
c. She has a higher incidence of having monozygotic twins because she is Caucasian.
d. If the ultrasound showed two separate placentas, the twins must be dizygotic.
e. If the ultrasound showed two separate placentas, the twins cannot be monozygotic.

The answer is b. The incidence of monozygotic twinning is constant at a rate of one


set per 250 births around the world. It is unaffected by race, heredity, age, parity, or
infertility agents. The incidence of dizygotic twinning is influenced by all of these
factors, and varies based on group. These twins of the same gender could be
monozygotic or dizygotic. Two identifiable chorions can occur in monozygotic or
dizygotic twinning. Dizygotic twins will always have two amnions and two chorions,
since they result from fertilization of two eggs. Therefore, dizygotic twins may be of
the same or different genders. The placentas of dizygotic twins may be totally
separate, or intimately fused, depending on the location of the implantation of the two
zygotes. Monozygotic twins are always of the same gender because they originate
from the division of one zygote; however, they may be monochorionic or dichorionic
depending on when the separation of the twins occurred. Twenty to thirty percent of
monozygotic twins have dichorionic, diamniotic placentation (similar to dizygotic
twins), which results from separation of the blastocyst within the first 72 hours after
fertilization. Division that occurs between days 4 and 8 will result in monochorionic,
diamniotic twins. One percent of monozygotic twins will be monochorionic,
monoamniotic, which occurs with division after day 8 but before the embryonic disc
is formed. Conjoined twins are always monozygotic, and occur with late division after
formation of the embryonic disk.

10. After delivery of a term newborn with Apgar scores of 2 at 1 minute and 7 at 5
minutes, you ask that blood from the umbilical arteries be collected for pH. The
umbilical arteries carry which of the following?
a. Oxygenated blood to the placenta
b. Oxygenated blood from the placenta
c. Deoxygenated blood to the placenta
d. Deoxygenated blood from the placenta
e. Mixed oxygenated blood from the placenta

The answer is c. Deoxygenated fetal blood is returned directly to the placenta through
the umbilical branches of the two hypogastric arteries. The umbilical arteries exit
through the abdominal wall at the umbilicus and continue by way of the umbilical
cord to the placenta. Deoxygenated blood circulates through the placenta then returns,
oxygenated, to the fetus via the umbilical vein. The umbilical arteries atrophy and
obliterate within 3 to 4 days after birth; remnants are called umbilical ligaments.

Questions 11 to 12

11. A 25-year-old P0 presents for routine anatomy ultrasound at 20 weeks’ gestation. The
only significant finding at the time of ultrasound is the presence of a single umbilical
artery (SUA). How should you counsel this patient about the finding of a SUA?
a. It is a very common finding and is insignificant.
b. Fetal karyotype should be determined, because this finding is associated with an
increased risk of aneuploidy.
c. Careful anatomic survey should be performed, because it is an indicator of an
increased incidence of congenital anomalies of the fetus.
d. It is equally common in newborns of diabetic and nondiabetic mothers.
e. Even if it is the only abnormality present, SUA is commonly associated with
adverse pregnancy outcomes.

The answer is c. The finding of a SUA occurs in approximately 1% of pregnancies,


and 5% of at least one twin. The incidence of SUA is increased in diabetic mothers.
The incidence of major fetal malformations when SUA is identified has been reported
to be as high as 18%, and usually involves the cardiac or renal systems; therefore, a
careful anatomic survey is indicated. The rate of aneuploidy in the setting of isolated
SUA is not increased, so routine karyotype analysis is not needed unless there are
other indications to offer this testing. In the absence of other findings, SUA is rarely
associated with poor pregnancy outcomes.

12. Targeted ultrasound does not demonstrate any other abnormalities. The patient asks
you if this SUA will impact how you manage the rest of her pregnancy. What should
you tell her?
a. Her pregnancy will not be managed any differently due to the isolated finding of
SUA.
b. The next step in her management should be genetic amniocentesis.
c. She will require periodic assessments of fetal growth.
d. She should be delivered by 39 weeks’ gestation at the latest.
e. She will require delivery by cesarean.
The answer is c. The finding of SUA in the absence of other abnormalities does not
require karyotype evaluation, early delivery, or delivery by cesarean. The timing and
mode of delivery may be determined by routine obstetric indications. Patients with a
fetus with SUA should undergo periodic growth assessments with ultrasound, as there
is an increased risk of growth restriction in these fetuses.

13. A 22-year-old G1P0 at 28 weeks’ gestation by LMP presents to labor and delivery
complaining of decreased fetal movement. She has had no prenatal care. On the fetal
monitor there are no contractions. The fetal heart rate is 150 beats per minute and
reactive, with no decelerations in the fetal heart tracing. An ultrasound demonstrates a
28-week fetus with normal anatomy and size consistent with menstrual dates. The
placenta is implanted on the posterior uterine wall and its margin is well away from
the cervix. A succenturiate lobe of the placenta is seen implanted low on the anterior
wall of the uterus. Doppler flow studies indicate a blood vessel is traversing the cervix
connecting the two lobes. This patient is most at risk for which of the following?
a. Premature rupture of the membranes
b. Fetal exsanguination after rupture of the membranes
c. Torsion of the umbilical cord caused by velamentous insertion of the umbilical cord
d. Amniotic fluid embolism
e. Placenta accreta

The answer is b. This patient has a vasa previa. When fetal vessels cross the internal
os (vasa previa), rupture of membranes may be accompanied by rupture of a fetal
vessel leading to fetal exsanguination. Vasa previa does not increase the risk for
placenta accreta or amniotic fluid embolism. With velamentous insertion of the cord,
the umbilical vessels separate in the membranes at a distance from the placental
margin which they reach surrounded only by amnion. Such insertion occurs in about
1% of singleton gestations but is quite common in multiple pregnancies. Fetal
malformations are more common with velamentous insertion of the umbilical cord.
An increased risk of premature rupture of membranes and torsion of the umbilical
cord has not been described in association with velamentous insertion of the cord.

14. A healthy 25-year-old G1P0 at 37 weeks’ gestational age comes to your office to see
you for a routine obstetric visit. She reports that on several occasions she has
experienced dizziness, light-headedness, and feeling as if she is going to pass out
when she lies down on her back to take a nap. What is the most appropriate plan of
management for this patient?
a. Perform an electrocardiogram
b. Monitor her for 24 hours with a Holter monitor to rule out an arrhythmia
c. Perform an arterial blood gas analysis
d. Refer her to a cardiologist
e. Reassure and encourage her not to lie flat on her back

The answer is e. Late in pregnancy, when the mother assumes the supine position, the
gravid uterus compresses the inferior vena cava and decreases venous return to the
heart. This results in decreased cardiac output and symptoms of dizziness, light-
headedness, and syncope. This significant arterial hypotension resulting from inferior
vena cava compression is known as supine hypotensive syndrome or inferior vena
cava syndrome. Therefore, it is not recommended that women remain in the supine
position for any prolonged period of time in the latter part of pregnancy. When
patients describe symptoms of the supine hypotensive syndrome, there is no need to
proceed with additional cardiac or pulmonary workup.

15. A 22-year-old primigravida presents to your office for a routine OB visit at 34 weeks’
gestational age. She voices concern because she has noticed an increasing number of
spidery veins appearing on her face, upper chest, and arms. She is upset with the
unsightly appearance of these veins and wants to know what you recommend to get
rid of them. How should you counsel this patient?
a. Tell her that this is a condition which requires evaluation by a vascular surgeon.
b. Tell her that you are concerned that she may have serious liver disease and order
liver function tests.
c. Tell her that you are going to refer her to a dermatologist for further workup and
evaluation.
d. Tell her that the appearance of these blood vessels is a normal occurrence with
pregnancy.
e. Tell her to wear an abdominal binder.

The answer is d. Vascular spiders, or angiomas, are common findings during


pregnancy. They form as a result of the hyper-estrogenism associated with normal
pregnancies and are of no clinical significance. The presence of these angiomas does
not require any additional workup or treatment, and they will resolve spontaneously
after delivery. Reassurance to the patient is all that is required.

16. You are the third year medical student assigned to labor and delivery. A 29-year-old
P3003 at 29 weeks with known placenta previa presents to the triage area with a
report of vaginal bleeding. The fetal heart tracing is reactive and the bleeding is
minimal. You take history and present her to your intern. You accompany the intern
to triage to further evaluate the patient together. Your intern confirms the history and
prepares to perform a digital cervical examination. What should your next step be in
this situation?
a. Watch the intern perform a digital cervical examination
b. Ask if you may also perform a digital cervical examination
c. Remind the intern that the patient has a placenta previa and should not have a
digital cervical examination
d. Suggest that the intern perform an ultrasound to evaluate the placenta
e. Do not say anything at the time, but afterward remind the intern that the patient has
a placenta previa and should not have a digital cervical examination

The answer is c. Patient safety has no hierarchy. Placenta previa is a condition where
the placenta is implanted over the internal cervical os. Digital cervical examination is
contraindicated in this setting due to the possibility of causing severe hemorrhage.
The correct next step is to speak up to make sure the intern knows that the patient has
a placenta previa and should not have a digital cervical examination.

17. A patient presents in labor at term. Clinical pelvimetry is performed. She has an oval-
shaped pelvis with the anteroposterior (AP) diameter at the pelvic inlet greater than
the transverse diameter. The baby is occiput posterior. The patient most likely has
what kind of pelvis?
a. A gynecoid pelvis
b. An android pelvis
c. An anthropoid pelvis
d. A platypelloid pelvis
e. An androgenous pelvis

The answer is c. By tradition, pelves are classified as belonging to one of four major
groups, based on the shape of the pelvis. A line drawn through the greatest diameter
of the pelvic inlet divides the pelvis into anterior and posterior sections, and the shape
of these segments helps determine the pelvis type. The gynecoid pelvis is the classic
female pelvis, with a posterior sagittal diameter of the inlet only slightly shorter than
the anterior sagittal diameter. The posterior pelvis is rounded and wide, the sidewalls
are straight, the spines are not prominent, and the pubic arch is wide. In the android
pelvis, the posterior sagittal diameter at the inlet is much shorter than the anterior
sagittal diameter, limiting the use of the posterior space by the fetal head. The
sidewalls are convergent, the spines are prominent, and the pubic arch is narrowed. In
the anthropoid pelvis, the AP diameter of the inlet is greater than the transverse
diameter, resulting in an oval with large sacrosciatic notches, convergent side walls,
prominent ischial spines, and a narrow pubic arch. The platypelloid pelvis is flattened
with a short AP and wide transverse diameter. Wide sacrosciatic notches are common.
The pelves of most women do not fall into a pure type and are blends of one or more
of these types.

18. Pelvic examination is performed in a 34-year-old P0101 at 34 weeks’ gestation who is


in labor. The patient is noted to be 6 cm dilated, and completely effaced with the fetal
nose and mouth palpable. The chin is pointing toward the maternal left hip. This is an
example of which of the following?
a. Transverse lie
b. Mentum transverse position
c. Occiput transverse position
d. Brow presentation
e. Vertex presentation.

The answer is b. The lie of the fetus refers to the relation of the long axis of the fetus
to that of the mother, and is classified as longitudinal, transverse, or oblique. The
presentation, or presenting part, refers to the portion of the baby that is foremost in the
birth canal. The presentation may be cephalic, breech, or shoulder. Cephalic
presentations are further classified as vertex, brow, or face. The position is the relative
relationship of the presenting part of the fetus to the mother. In this instance, the fetus
is cephalic, with the face presenting. In a face presentation, the fetal head is
hyperextended so that the occiput is in contact with the fetal back, and the chin
(mentum) is presenting. The mentum is the point of reference of the fetus when
describing the position of the face. Since the mentum is pointing toward the mother’s
left hip, the fetal position is described as mentum transverse. In vertex presentations,
the occiput is the point of reference for determining position. In breech presentations,
the sacrum is the point of reference.

19. You are counseling a 36-year-old obese, Hispanic G2P1 at 36 weeks’ gestation about
route of delivery. During her first pregnancy, she was induced at 41 weeks’ gestation
for mild preeclampsia, and delivered by cesarean as a result of fetal distress during
her induction. The patient would like to know if she can have a trial of labor after
cesarean (TOLAC) with this pregnancy. Which of the following is the best response
to this patient?
a. No, since she has never had a vaginal delivery.
b. Yes, but only if she had a low transverse uterine incision.
c. No, because once she has had a cesarean delivery, she must deliver all of her
subsequent children by cesarean.
d. Yes, but only if her skin incision was a Pfannensteil.
e. Yes, but she must wait until she goes into labor spontaneously to have a repeat
cesarean.

The answer is b. A patient with a prior low transverse incision may attempt a
TOLAC. Repeat cesarean and TOLAC both have inherent risks. The main risk of
TOLAC that increases maternal and neonatal morbidity is uterine rupture, the risk of
which is impacted significantly by the location of the uterine incision. A low
transverse incision is made transversely through the lower uterine segment, which
does not actively contract during labor. The risk of uterine rupture after prior low
transverse incision is less than 1%. The skin incision does not reflect the location of
the uterine incision, and therefore is not an indicator of the suitability of TOLAC for a
patient. Although a prior vaginal delivery increases the success rate for a successful
VBAC, a prior vaginal birth is not a prerequisite for a TOLAC. If the patient desires a
repeat cesarean delivery, this should be performed at 39 weeks as a scheduled
procedure.

20. The patient wants to know about the probability of success if she chooses to undergo
TOLAC. What can you tell her about factors that impact the probability of success in
TOLAC?
a. The probability of successful TOLAC is increased for her because she is Hispanic.
b. She is likely to have a successful TOLAC because she has never had a vaginal
delivery.
c. Her weight does not impact her chance for successful TOLAC.
d. Her age does not impact her chance for successful TOLAC.
e. If she goes into labor spontaneously before 40 weeks, her chance for successful
TOLAC will be increased.
The answer is e. Good candidates for TOLAC are women in whom the balance of
risks and benefits are acceptable to the patient and health care provider. Decisions
regarding TOLAC must be made on an individual basis while taking these factors into
account. Most evidence suggests that most women with one prior low transverse
cesarean should be counseled about vaginal birth after cesarean (VBAC) and offered
TOLAC. Factors that increase the probability of success include prior vaginal delivery
and spontaneous labor. Factors that predict a decreased probability of success include
increased maternal age, Hispanic or African American ethnicity, postdates gestation,
and maternal obesity. Therefore, spontaneous labor prior to 40 weeks would provide
this patient with the greatest chance of successful TOLAC.

21. The patient has still not gone into spontaneous labor at 41 weeks’ gestation. You see
her in clinic and her blood pressure is 150/90 mmHg and she has +3 proteinuria on
urine dipstick. You send her to labor and delivery for further evaluation, and her
blood pressure remains elevated, consistent with a diagnosis of preeclampsia. You
examine her cervix and find that it is closed and thick. She asks whether she can
undergo induction of labor at this point. What should you tell her about induction of
labor?
a. She may be induced after using a prostaglandin as a cervical ripening agent.
b. Her chance of successful VBAC is just as high with induction of labor as it is with
spontaneous labor.
c. Prior cesarean delivery is a contraindication to induction of labor.
d. She may be induced with a mechanical cervical ripening agent such as a
transcervical catheter.
e. Her unfavorable cervical exam does not impact her chance of successful TOLAC.

The answer is d. Induction of labor for maternal or fetal indications is an option for
women undergoing TOLAC. However, the chance of successful TOLAC with
induction versus spontaneous labor is lower, and her unfavorable cervical
examination decreases the potential success rate. Data support use of mechanical
ripening agents, such as transcervical catheter, in this setting of an unfavorable cervix.
Prostaglandins are not used for cervical ripening due to concerns over increased risk
of uterine rupture.

22. A 32-year-old poorly controlled diabetic G2P1 is undergoing amniocentesis at 38


weeks for fetal lung maturity prior to having a repeat cesarean delivery. Which of the
following laboratory tests results on the amniotic fluid would best indicate that the
fetal lungs are mature?
a. Phosphatidylglycerol (PG) is absent
b. Lecithin/sphingomyelin (L/S) ratio of 1:1
c. Lecithin/sphingomyelin ratio of 1.5:1
d. Lecithin/sphingomyelin ratio of 2.0:1
e. Phosphatidylglycerol is present

The answer is e. The lecithin-to-sphingomyelin (L/S) ratio in amniotic fluid is close to


1 until about 34 weeks of gestation, when the concentration of lecithin begins to rise.
For pregnancies of unknown duration but otherwise uncomplicated, the risk of
respiratory distress syndrome (RDS) is relatively low when the L/S is at least 2:1.
Maternal hypertensive disorders and fetal growth retardation may accelerate the rate
of fetal pulmonary maturation, possibly as a result of chronic fetal stress. A delay in
fetal pulmonary maturation is observed in pregnancies complicated by maternal
diabetes or erythroblastosis fetalis. A risk of RDS of 40% exists with an L/S ratio of
1.5:2; when the L/S ratio is less than 1.5, the risk of RDS is 73%. When the L/S ratio
is greater than 2, the risk of RDS is slight. However, when the fetus is likely to have a
serious metabolic compromise at birth (eg, diabetes or sepsis), RDS may develop
even with a mature L/S ratio (> 2.0). This may be explained by lack of PG, a
phospholipid that enhances surfactant properties. The identification of PG in amniotic
fluid provides considerable reassurance (but not an absolute guarantee) that RDS will
not develop. Moreover, contamination of amniotic fluid by blood, meconium, or
vaginal secretions will not alter PG measurements.

23. A 26-year-old G1P0 patient at 34 weeks’ gestation is being evaluated with Doppler
ultrasound studies of the fetal umbilical arteries. The patient is a healthy smoker. Her
fetus has shown evidence of intrauterine growth restriction (IUGR) on previous
ultrasound examinations. The Doppler studies currently show that the systolic to
diastolic ratio (S/D) in the umbilical arteries is much higher than it was on her last
ultrasound 3 weeks ago, and there is now reverse diastolic flow. Which of the
following is correct information to share with the patient?
a. The Doppler studies indicate that the fetus is doing well.
b. With advancing gestational age, the S/D ratio is expected to rise.
c. These Doppler findings are normal in someone who smokes.
d. Reverse diastolic flow is normal as a patient approaches full term.
e. The Doppler studies are worrisome, and indicate that the fetal status is
deteriorating.

The answer is e. Simple continuous-wave Doppler ultrasound can be used to display


flow velocity waveforms as a function of time. With increased gestational age, in
normal pregnancy there is an increase in enddiastolic flow velocity relative to peak
systolic velocity, which causes the S/D ratio to decrease with advancing gestation. An
increase in S/D ratio is associated with increased resistance in the placental vascular
bed, as can be noted in preeclampsia or fetal growth restriction. Nicotine and maternal
smoking have also been reported to increase the S/D ratio. Many studies document the
value of umbilical Doppler flow studies in recognition of fetal compromise. The S/D
ratio increases as the fetal condition deteriorates; this is most severe in cases of absent
or reversed end diastolic flow.

24. A 17-year-old primipara presents to your office at 41 weeks. Her pregnancy has been
uncomplicated. Because her cervix is unfavorable for induction of labor, she is being
followed with biophysical profile (BPP) testing. Which of the following is correct
information to share with the patient regarding BPPs?
a. BPP testing includes assessment of amniotic fluid volume, fetal breathing, fetal
body movements, fetal body tone, and contraction stress testing.
b. The false-negative rate of the BPP is 10%, so a reassuring BPP should be repeated
in 48 hours.
c. False-positive results on BPP are rare even if the amniotic fluid level is low.
d. Spontaneous decelerations during BPP testing are associated with significant fetal
morbidity.
e. A normal BPP should be repeated twice a week.

The answer is d. The BPP is based on FHR monitoring with nonstress test (NST) in
addition to four parameters observed on real-time ultrasonography—amniotic fluid
volume, fetal breathing, fetal body movements, and fetal body tone. Each parameter
gets a score of 0 or 2. A score of 8 or 10 is considered normal, a score of 6 is
equivocal, and a score of 4 or less is abnormal and prompts delivery. The false-
negative rate for the BPP is less than 0.1%, but false-positive results are relatively
frequent, with poor specificity. Oligohydramnios is an ominous sign, as are
spontaneous

25. A 32-year-old G2P1 at 28 weeks’ gestation presents to labor and delivery with a chief
complaint of vaginal bleeding. Her vital signs are— blood pressure 115/67 mm Hg,
pulse 87 beats per minute, temperature 37.0°C, respiratory rate 18 breaths per minute.
She reports no contraction and states that the baby is moving normally. On
ultrasound, the placenta is located on the anterior wall of the uterus, and completely
covers the internal cervical os. Which of the following would most increase her risk
for hysterectomy?
a. Desire for sterilization
b. Development of disseminated intravascular coagulopathy (DIC)
c. Placenta accreta
d. Prior vaginal delivery
e. Smoking

The answer is c. Prior cesarean delivery and placenta previa, especially with an
anteriorly located placenta, increase the risk of placenta accreta, increta, and percreta.
These are situations where the placenta is abnormally adherent to the uterine wall. In
placenta accreta, the placental villi are abnormally attached to the myometrium. In
placenta increta, the villi invade into the myometrium, and in placenta percreta, the
villi penetrate through the myometrium. Placenta accreta, increta, or percreta typically
require treatment with hysterectomy. The incidence of these disorders has increased
due to the increased cesarean delivery rate. Placenta accrete may be suspected on
ultrasound, but MRI is often required to confirm the diagnosis. Advanced maternal
age, multiparity, prior cesarean delivery, and smoking are all risk factors for placenta
previa. Painless bleeding is the most common symptom, and is rarely fatal. Vaginal
examination to evaluate for placenta previa is contraindicated, unless the woman is in
the operating room prepared for immediate cesarean delivery, because even the most
gentle examination can cause significant hemorrhage. Vaginal examinations are rarely
necessary, because ultrasound is usually readily available to make the diagnosis of
placenta previa. Cesarean delivery is necessary in essentially all cases of placenta
previa. Because of the poor contractile nature of the lower uterine segment,
uncontrollable hemorrhage may follow removal of the placenta. Hysterectomy may be
indicated if conservative methods to control hemorrhage fail. Resuscitation with
blood products is the treatment for disseminated intravascular coagulopathy, not
hysterectomy. Sterilization itself is not an indication for hysterectomy at the time of
cesarean delivery, because the complications of surgery are much increased with a
cesarean hysterectomy.

26. A patient at 17 weeks’ gestation is diagnosed with an intrauterine fetal demise. She
desires expectant management. She returns to your office 5 weeks later, and her vital
signs are—blood pressure 110/72 mm Hg, pulse 93 beats per minute, temperature
36.38°C, respiratory rate 16 breaths per minute. She has not had a miscarriage,
although she has had some occasional spotting. Her cervix is closed on examination.
This patient is at increased risk for which of the following?
a. Septic abortion
b. Recurrent abortion
c. Consumptive coagulopathy
d. Future infertility
e. Ectopic pregnancies

The answer is c. In women with intrauterine fetal demise, labor usually occurs within
2 weeks. Women are typically offered expectant management versus active
management with surgical or medical evacuation of the uterus. If the fetus is retained
longer than 1 month, 25% of women can develop coagulopathy, which is manifested
by decreased fibrinogen, elevated fibrin degradation products, and decreased platelets.
Septic abortions were more frequently seen during the era of illegal abortions,
although occasionally sepsis can occur if there is incomplete evacuation of the
products of conception in either a therapeutic or spontaneous abortion. However,
since her cervix is closed and no tissue has passed, septic abortion is unlikely.
Intrauterine fetal demise has no impact on future fertility or association with ectopic
pregnancies.

Question 27 - 28

27. A 24-year-old G1P0 presents at 30 weeks’ gestation for a new OB visit. She provides
you with the official report of a dating ultrasound performed at 12 weeks; however,
shortly thereafter, she moved out of state and has not had prenatal care. She has no
medical problems, and has a normal BMI. She reports some abdominal cramping and
shortness of breath. During her visit, you examine her cervix and it is closed. You
measure her fundal height at 50 cm. 108. What is the next best step in management?
a. Order an ultrasound
b. Tell the patient that she is most likely having twins
c. Teach her how to do fetal kick counts, and instruct her to return in 1 week
d. Tell her that her baby will be very large and recommend a caesarean delivery
e. Order a glucose tolerance test 109.
The answer is a. This patient has an abnormally large fundal height. A fundal height
should typically measure within 3 cm of the patient’s gestational age. In this case, the
gestational age is known because the patient had an early ultrasound, and was able to
produce the report for confirmation. Given this early ultrasound, it is unlikely that the
patient is having twins, as this would have been identified at her 12-week ultrasound.
Gestational diabetes is sometimes associated with polyhydramnios, and the patient
does need a glucose tolerance test as part of her prenatal care, but this is not the next
best step. It is possible she will have a large baby, but this alone would not explain the
size greater than dates measurement. A fundal height of 50 cm requires further
investigation, and the best next step is ultrasound.

28. An ultrasound is performed, and demonstrates a singleton fetus with an estimated


fetal weight (EFW) in the 53 percentile. The amniotic fluid index is 30 cm, consistent
with a diagnosis of polyhydramnios. How should you counsel this patient?
a. She does not require any further evaluation.
b. The incidence of associated malformations is approximately 3%.
c. Maternal edema, especially of the lower extremities and vulva, is rare.
d. Esophageal atresia is accompanied by polyhydramnios in nearly 10% of cases.
e. Potential complications include placental abruption, uterine dysfunction, and
postpartum hemorrhage.

The answer is e. Polyhydramnios is defined as an excessive quantity of amniotic fluid,


and occurs in 1% to 2% of pregnant women. When diagnosed, it requires further
evaluation for genetic or anatomic anomalies, diabetes, and infection, although
approximately 40% of cases are determined to be idiopathic. The incidence of
associated malformations is about 20%, with CNS and GI abnormalities being
particularly common, due to their impact on swallowing and absorption of amniotic
fluid. Polyhydramnios accompanies about half of cases of anencephaly, and nearly all
cases of esophageal atresia. Edema of the lower extremities, vulva, and abdominal
wall is common, and results from compression of major venous systems. The most
frequent maternal complications are placental abruption, uterine dysfunction, and
postpartum hemorrhage

29. During routine ultrasound surveillance of a twin pregnancy, twin A weighs 1200 g
and twin B weighs 750 g. Polyhydramnios is noted around twin A, while twin B has
oligohydramnios. Which of the following statements correctly describes this
syndrome?
a. The donor twin develops polyhydramnios more often than the recipient twin.
b. Gross differences may be observed between donor and recipient placentas.
c. The donor twin usually suffers from a hemolytic anemia.
d. The donor twin is more likely to develop widespread thromboses.
e. The donor twin often develops polycythemia.

The answer is b. In the twin-to-twin transfusion syndrome (TTTS), the donor twin is
always anemic. This is not due to a hemolytic process, but rather to the direct transfer
of blood to the recipient twin, who becomes polycythemic. The recipient may suffer
thromboses secondary to hypertransfusion and subsequent hemoconcentration.
Although the donor placenta is usually pale and somewhat atrophied, that of the
recipient is typically congested and enlarged. Polyhydramnios can develop in either
twin, but is more frequent in the recipient twin due to circulatory overload. When
polyhydramnios occurs in the donor, it is due to congestive heart failure caused by
severe anemia.

30. A 32-year-old G5P1 presents for her first prenatal visit. A complete obstetrical,
gynecological, and medical history and physical examination is performed. Which of
the following would be an indication for elective cerclage placement?
a. Three spontaneous first-trimester abortions
b. Twin pregnancy
c. Three second-trimester pregnancy losses without evidence of labor or abruption
d. History of loop electrosurgical excision procedure for cervical dysplasia
e. Cervical length of 35 mm by ultrasound at 18 weeks

The answer is c. Cervical insufficiency (or incompetence) describes the inability of


the cervix to retain a pregnancy in the absence of contractions (or labor) in the second
trimester. It is diagnosed based on a history of painless cervical dilation after the first
trimester with delivery usually before 24 weeks, without contractions or other clear
pathology (ie, infection, ruptured membranes). Based on current data, a shortened
cervical length on ultrasound is associated with an increased risk of preterm birth, but
is not sufficient for the diagnosis of cervical incompetence. Cerclage is indicated in a
patient with a history of one or more second-trimester losses related to cervical
incompetence. Cerclage is not indicated for the prevention of first-trimester losses,
nor has it been shown to improve the preterm delivery rate or neonatal outcome in
twin gestations. Evidence is currently lacking for the benefit of cerclage solely due to
a history of prior loop electrosurgical excision procedure or cone biopsy. Serial
transvaginal ultrasound evaluation of cervical length can be considered in women
with a history of second and early third-trimester deliveries. A cervical length less
than 25 mm or funneling of more than 25% or both is associated with an increased
risk of preterm delivery.

31. 146. A 33-year-old G3P2 at 38 weeks’ gestation develops flu-like illness and breaks
out with a pruritic, vesicular lesions all over her body. Three days later she goes into
spontaneous labor and delivers a healthy appearing male infant via vaginal delivery.
Her lesions are beginning to heal and she feels well. What is the most appropriate next
step in the management of this patient and her baby?
a. Administer intravenous acyclovir to the mother
b. Administer intravenous acyclovir to the baby
c. Administer varicella-zoster immune globulin to the baby
d. Administer varivax (varicella vaccine) to the baby
e. Administer zostavax (herpes zoster vaccine) to the mother
The answer is c. Varicella, or chicken pox, is usually diagnosed based on the clinical
findings of a classic pruritic, vesicular rash. Pregnant women should have varicella
immunity documented in early pregnancy by a history of previous infection or
varicella vaccination. Pregnant women who have no history of chicken pox or have
serology demonstrating lack of immunity should avoid varicella infected individuals
until their lesions have crusted over and they are no longer infectious. Neonatal
mortality rates are close to 25% when maternal varicella develops around the time of
delivery, due to the lack of protective maternal antibodies and the relative immaturity
of the fetal immune system. Therefore, if a mother has clinical evidence of varicella
infection 5 days before or up to 48 hours after delivery, the newborn should receive
varicella-zoster immune globulin. Typically, varicella infection in the mother only
requires supportive therapy, but pregnant women have a higher and mortality related
to development of pneumonia. If pneumonia is diagnosed, intravenous acyclovir
should be given. The newborn should be isolated from the mother if she is infective,
and if the neonate develops signs or symptoms of varicella infection, then intravenous
acyclovir would be administered. Pregnant women should not receive the live-
attenuated varicella vaccine.

32. A 29-year-old G1 at 9 weeks’ gestation presents to your office for a new OB visit.
She reports a history of well-controlled hypothyroidism. She takes 88 mcg of
levothyroxine daily. How do you expect her thyroid laboratory values to change
during pregnancy?
a. The thyroid-stimulating hormone (TSH) and free T4 will not change during
pregnancy.
b. The TSH will increase and the free T4 will decrease.
c. The TSH will increase and there will be no change in the free T4.
d. The TSH and the free T4 will increase.
e. The free T4 will not change, and the change in TSH will vary by trimester

The answer is e. There are considerable changes in maternal thyroid function during
pregnancy. Maternal total or bound thyroid hormone levels increase with serum
concentration of thyroid-binding globulin. TSH decreases in early pregnancy because
of weak stimulation of its receptors by human chorionic gonadotropin (hCG) during
the first trimester After the first trimester, TSH levels return to baseline values and
progressively increase in the third trimester related to placental growth and production
of placental deiodinase. Free T4 remains stable during pregnancy. A high TSH and
low free T4 are characteristic of overt hypothyroidism. These physiologic changes
should be considered when interpreting thyroid function test results during pregnancy.

33. A 19-year-old P0 presents for her first OB visit at 10 weeks’ gestation. You order
routine OB laboratory tests, and it returns showing a positive nucleic acid probe for
Neisseria gonorrhoeae. One year ago, she was treated with ampicillin for a simple
urinary tract infection and developed a severe allergic reaction. Which of the
following is the best option for treatment at this time?
a. Tetracycline
b. Doxycycline
c. Azithromycin
d. Ceftriaxone
e. Penicillin

The answer is c. Patients with a severe allergic reaction to ampicillin should not
receive penicillin. Patient with this type of reaction have up to a 20% incidence of
reaction to cephalosporins, so ceftriaxone should be avoided as well unless
desensitization is undertaken. Spectinomycin used to be the treatment of choice for
pregnant women with Neisseria gonorrhoeae infections and who were allergic to
penicillin; however, the production of this medication was discontinued in the United
States in 2006. The use of doxycycline or tetracycline is generally contraindicated in
pregnancy. Azithromycin 2 g orally as a single dose may be used as an alternative to
treat both gonorrhea and chlamydia.

34. A 22-year-old pregnant woman has just been diagnosed with toxoplasmosis. Which of
the following risk factors is most likely to have contributed to her diagnosis?
a. Eating raw meat
b. Eating raw fish
c. Owning a dog
d. English nationality
e. Having viral infections in early pregnancy

The answer is a. Toxoplasmosis is caused by the intracellular parasite Toxoplasma


gondii. This infection is usually asymptomatic and self limited, but can present with
asymptomatic cervical lymphadenopathy, fever, malaise, night sweats, and myalgias.
Symptoms occur in only 10% to 20% of immunocompetent adults. Human infection
can result from ingestion of raw or under-cooked meat infected by the organism, or
from contact with infected cat feces. The French, because their diet includes raw
meat, have a higher incidence (but not the English). The incidence of vertical
transmission through the placenta varies by trimester, with the highest risk of
transmission in the third trimester. The earlier the fetus is infected, the more severe
the disease.

35. A 17-year-old woman at 22 weeks’ gestation presents to the emergency department


with a 3-day history of nausea, vomiting, and abdominal pain. The pain started in the
middle of the abdomen, and is now located along her mid-to-upper right side. She is
noted to have a temperature of 38.4°C (101.1°F). She reports no prior medical
problems or surgeries. How does pregnancy alter the diagnosis and treatment of the
disease?
a. Owing to anatomical and physiological changes in pregnancy, diagnosis is easier to
make.
b. Surgical treatment should be delayed since the patient is pregnant.
c. Fetal outcome is improved with delayed diagnosis.
d. The incidence is unchanged in pregnancy.
e. The incidence is higher in pregnancy.
The answer is d. The incidence of appendicitis in pregnancy is 1 in 2000, the same as
that in the nonpregnant population. The diagnosis can be difficult to make during
pregnancy because leukocytosis, nausea, and vomiting are common in pregnancy. In
addition, the upward displacement of the appendix by the uterus may cause
appendicitis to have a nonclassic presentation. Surgery is necessary even if the
diagnosis is not certain. Rupture of the appendix is more likely in pregnant women,
likely due to the delay in diagnosis and reluctance to operate on pregnant women.

36. A 24-year-old P1001 presents at 8 weeks’ gestation and reports a history of


pulmonary embolism 3 years ago during her first pregnancy. She was treated with
intravenous heparin followed by several months of oral warfarin (coumadin) and has
had no further evidence of thromboembolic disease. How should her current
pregnancy be managed?
a. Since she has had no further events or problems for 3 years, her risk of
thromboembolism is no longer increased, and she does not require therapy during this
pregnancy.
b. Because she has had no problems for 3 years, she may be treated only with a baby
aspirin daily.
c. She should be managed with Doppler ultrasonography of the bilateral lower
extremities once per trimester to screen for deep vein thrombosis.
d. The patient should be placed on low-dose unfractionated heparin therapy or low
molecular weight heparin therapy throughout pregnancy and puerperium.
e. She only requires anticoagulation during the third trimester.

The answer is d. Pregnancy is considered a hypercoaguable state. Patients with a


history of thromboembolic disease in pregnancy are at high risk of developing it in
subsequent pregnancies, and therefore should be anticoagulated. Baby aspirin is not
considered adequate treatment. Pregnant patients with a history of venous
thromboembolism should be treated with either low-dose unfractionated heparin
therapy or low molecular weight heparin therapy during the pregnancy and through
the postpartum period, as this is the time of highest risk of clot formation. Doppler
ultrasonography is the most common way to diagnose a deep vein thrombosis, but is
not considered a screening test, and should not be ordered each trimester in the
absence of clinical symptoms or signs.

37. A 29-year-old G3P2 black woman in the 33 week of gestation is admitted to the
emergency room because of acute abdominal pain that has been increasing during the
past 24 hours. The pain is severe and is radiating from the epigastrium to the back.
The patient has vomited a few times and has not eaten or had a bowel movement since
the pain started. On examination, you observe an acutely ill patient lying on the bed
with her knees drawn up. Her blood pressure is 100/70 mm Hg, her pulse is 110 beats
per minute, and her temperature is 38.8°C (101.8°F). On palpation, the abdomen is
somewhat distended and tender, mainly in the epigastric area, and the uterine fundus
reaches 31 cm above the symphysis. Hypotonic bowel sounds are noted. Fetal
monitoring reveals a normal pattern of fetal heart rate (FHR) without uterine
contractions. On ultrasonography, the fetus is in vertex presentation and appropriate
in size for gestational age; fetal breathing and trunk movements are noted, and the
volume of amniotic fluid is normal. The placenta is located on the anterior uterine
wall and no previa is seen. Laboratory values show mild leukocytosis (12,000 cells
per mL); a hematocrit of 43%; mildly elevated serum glutamicoxaloacetic
transaminase (SGOT), serum glutamic-pyruvic transaminase (SGPT), and bilirubin;
and serum amylase of 180 U/dL. Urinalysis is normal. Which of the following is the
most likely diagnosis?
a. Acute degeneration of uterine leiomyoma
b. Acute cholecystitis
c. Acute pancreatitis
d. Acute appendicitis
e. Severe preeclamptic toxemia

The answer is c. The most probable diagnosis in this case is acute pancreatitis. The
pain caused by a myoma in degeneration is more localized to the uterine wall. Low-
grade fever and mild leukocytosis may appear with a degenerating myoma, but liver
function tests are usually normal. The other obstetrical causes of epigastric pain, such
as preeclampsia may exhibit disturbed liver function (sometimes associated with the
hemolysis, elevated liver enzymes, low platelets (HELLP) syndrome), but this patient
has only mild elevation of blood pressure and no proteinuria. Acute appendicitis in
pregnancy is one of the more common nonobstetric causes of abdominal pain.
Symptoms of acute appendicitis in pregnancy are similar to those in nonpregnant
patients, but the pain is more vague and poorly localized and the point of maximal
tenderness moves to the right upper quadrant with advancing gestation. Liver function
tests are normal with acute appendicitis. Acute cholecystitis may cause fever,
leukocytosis, and pain of the right upper quadrant with abnormal liver function tests,
but amylase levels would be elevated only mildly, if at all, and pain would be less
severe than described in this patient. The diagnosis that fits the clinical description
and the laboratory findings is acute pancreatitis. This disorder may be more common
during pregnancy, with an incidence of 1 in 100 to 1 in 10,000 pregnancies.
Cholelithiasis, chronic alcoholism, infection, abdominal trauma, some medications,
and pregnancy-induced hypertension are known predisposing factors. Leukocytosis,
hemoconcentration, and abnormal liver function tests are common laboratory findings
in acute pancreatitis. However, the most important laboratory finding is an elevation
of serum amylase levels, which appears 12 to 24 hours after onset of clinical disease.
Values may exceed 200 U/dL (normal values are 50 U/dL to 160 U/dL). Treatment
considerations for the pregnant patient with acute pancreatitis are similar to those in
nonpregnant patients. Intravenous hydration, nasogastric suction, enteric rest, and
correction of electrolyte imbalance and of hyperglycemia are the mainstays of therapy

38. An 18-year-old G1 is diagnosed with asymptomatic bacteriuria (ASB) at her first


prenatal visit at 15 weeks’ gestation, based on a urine culture performed as part of her
routine new OB laboratory findings. What is the next step in management?
a. Because she is asymptomatic, she does not require treatment.
b. She will only require treatment for ASB if she has sickle cell trait.
c. She only requires treatment if the culture is positive for group B streptococcus.
d. Twenty-five percent of women with ASB subsequently develop an acute
symptomatic urinary infection during the same pregnancy, and therefore she should
be treated with antibiotics.
e. She does not require treatment because ASB is not associated with adverse
pregnancy outcomes.

The answer is d. The term ASB is used to indicate persistent, actively multiplying
bacteria within the urinary tract without symptoms of a urinary infection. The
reported prevalence during pregnancy varies from 2% to 7%. The highest incidence
has been reported in black multiparas with sickle cell trait and the lowest incidence
among white women of low parity. In women who demonstrate ASB, the bacteriuria
is typically present at the time of the first prenatal visit; after an initial negative
culture of the urine, fewer than 1% develop a urinary infection. If ASB is not treated
during pregnancy, approximately 25% of infected women develop an acute infection.
Untreated ASB has been associated with an increase in complications such as low
birth weight, preterm birth, and pyelonephritis.

39. A 20-year-old G1 at 18 weeks of gestation is hospitalized for intravenous antibiotics


for the treatment of acute pyelonephritis. She develops shortness of breath and is
found to have tachypnea and decreased oxygen saturation. Chest x-ray reveals
pulmonary infiltrates consistent with pulmonary edema. What is the most likely cause
of this complication?
a. Acute renal failure
b. Allergic reaction
c. Bacteremia
d. Endotoxin release
e. Intravenous hydration

The answer is d. Endotoxin release can cause alveolar injury and lead to pulmonary
edema and acute respiratory distress. Endotoxin release can also cause renal
dysfunction manifested as increase serum creatinine, but this effect is usually
reversible with fluid resuscitation. Uterine contractions and hemolytic anemia are also
effects of endotoxin release. Bacteremia can be found in up to 20% of women with
pyelonephritis, but it is the endotoxin release that leads to alveolar damage. While
allergic reactions to antibiotics can cause respiratory symptoms, they do so by causing
bronchoconstriction. Intravenous hydration to ensure adequate urinary output (> 50
mL/h) is the mainstay of therapy. Careful monitoring of the input and output of the
patient is necessary so that fluid overload will not compound the pulmonary effects of
the endotoxin.

Question 40 - 42

A 30-year-old G1 at 6 weeks’ gestation by last menstrual period presents for prenatal


care. Her past medical history is significant for type 1 diabetes, which was diagnosed
at the age of 14.
40. What should you tell her about her insulin requirements during pregnancy?
a. Her insulin requirement will not change during pregnancy.
b. She will require less insulin due to increased sensitivity to insulin during
pregnancy.
c. She will require less insulin during pregnancy because she will experience
decreased insulin resistance.
d. As long as her glycosylated hemoglobin A1c (Hb A1c) is less than 6%, she will not
require any changes in her insulin management during pregnancy.
e. She should expect her insulin requirement to increase throughout the pregnancy.

The answer is e. Pregnancy is characterized by both increased insulin resistance and


decreased sensitivity to insulin. The increased insulin resistance is largely due to
placental hormones such as human placental lactogen, progesterone, and cortisol. The
management of type 1 diabetes in pregnancy focuses on glucose control, maximizing
diet, engaging in exercise, and insulin therapy. Insulin requirements will increase
during pregnancy, most markedly during the period between 28 and 32 weeks’
gestation.

41. Which of the following is the most common birth defect associated with diabetes?
a. Anencephaly
b. Encephalocele
c. Meningomyelocele
d. Sacral agenesis
e. Ventricular septal defect

The answer is e. Major congenital anomalies are the leading cause of perinatal
mortality in pregnancies complicated by type 1 diabetes, occurring in 6% to 12% of
infants of women with diabetes. It is believed that the increased risk of congenital
anomalies is a consequence of poor glucose control in the preconception and early
pregnancy period. Glycosylated hemoglobin (Hgb A1c) level correlates directly with
the frequency of anomalies. A HbA1c around 6% is associated with a fetal anomaly
rate close to that of the general population (2%-3%), whereas an HbA1c of 10% is
associated with a fetal anomaly rate of 20%-25%. The most common single organ
system anomalies are complex cardiac (38%), musculoskeletal (15%), and central
nervous system (anencephaly and/or spina bifida) (10%). Sacral agenesis is a rare
malformation seen in severely diabetic women.

42. Which of the following diabetic complications is most likely to be permanently


worsened by pregnancy?
a. Coronary artery disease (CAD)
b. Gastroparesis
c. Nephropathy
d. Neuropathy
e. Proliferative retinopathy
The answer is e. Pregnancy has been associated with exacerbation of many diabetes-
related complications. The rapid institution of strict glycemic control in women with
diabetes during pregnancy has been associated with acute worsening of diabetic
proliferative retinopathy, especially in women with coexisting hypertensive disorders.
Pregnant women with diabetes should undergo a complete eye examination at the
beginning of pregnancy, and should be monitored closely throughout. Most studies
have failed to demonstrate permanent worsening in renal function in women with
mild to moderate diabetic nephropathy. CAD does not necessarily worsen during
pregnancy, but in women with preexisting symptomatic CAD, the pregnancy-
associated hemodynamic changes may lead to an increased risk of myocardial
infarction and death. Diabetic neuropathy is not very well studied during pregnancy,
but may manifest as gastroparesis causing intractable nausea and vomiting.

43. A 33-year-old woman at 10 weeks’ gestation presents for her first prenatal visit.
Routine laboratory findings are drawn, and her hepatitis B surface antigen is positive.
Liver function tests are normal and her hepatitis B core and surface antibody tests are
negative. Which of the following is the best way to prevent neonatal infection?
a. Provide immune globulin to the mother
b. Provide hepatitis B vaccine to the mother
c. Perform a cesarean delivery at term
d. Provide hepatitis B vaccine to the neonate
e. Provide immune globulin and the hepatitis B vaccine to the neonate

The answer is e. Hepatitis B is transmitted by parenteral and sexual contact. Women


with multiple sex partners, those who engage in intravenous drug use, and those who
have sexual partners who engage in these risky behaviors are at highest risk to acquire
this infection. Infection of the newborn whose mother chronically carries the hepatitis
B virus can usually be prevented by the administration of hepatitis B immune globulin
very soon after birth, followed promptly by the hepatitis B vaccine.

44. A 38-year-old G1P0 presents to the obstetrician’s office at 37 weeks’ gestation


complaining of a rash on her abdomen that is becoming increasingly pruritic. The rash
started on her abdomen, and is starting to spread downward to her thighs. She reports
no previous history of any skin disorders or problems, and she reports no malaise or
fever. On physical examination, she is afebrile and her physician notes that her
abdomen, and most notably her stretch marks, is covered with red papules and
plaques. No excoriations or bullae are present. The patient’s face, arms, and legs are
unaffected by the rash. Which of the following is this patient’s most likely diagnosis?
a. Herpes gestationis
b. Pruritic urticarial papules and plaques of pregnancy (PUPPP)
c. Prurigo gravidarum
d. Intrahepatic cholestasis of pregnancy
e. Impetigo herpetiformis

The answer is b. PUPPP is the most common dermatologic condition of pregnancy. It


is more common in nulliparous women and occurs most often in the second and third
trimesters of pregnancy. PUPPP is characterized by erythematous papules and plaques
that are intensely pruritic and appear first on the abdomen. The lesions then
commonly spread to the buttocks, thighs, and extremities with sparing of the face.
Herpes gestationis is a blistering skin eruption that occurs more commonly in
multiparous patients in the second or third trimester of pregnancy. The presence of
vesicles and bullae help differentiate this skin condition from PUPPP.
Immunologically, it is indistinguishable from bullous pemphigoid. Prurigo gestationis
is a very rare dermatosis of pregnancy that is characterized by small, pruritic
excoriated lesions that occur between 25 and 30 weeks. The lesions first appear on the
trunk and forearms and can spread throughout the body as well. In cholestasis of
pregnancy, bile acids are cleared incompletely and accumulate in the dermis, which
causes intense itching. These patients develop pruritus in late pregnancy; there are no
characteristic skin changes or rashes except in women who develop excoriations from
scratching. Impetigo herpetiformis is a rare pustular eruption that forms along the
margins of erythematous patches. This skin condition usually occurs in late
pregnancy. The skin lesions usually begin at points of flexure and extend peripherally;
mucous membranes are commonly involved. Patients with impetigo herpetiformis
usually do not have intense pruritus, but more commonly have systemic symptoms of
nausea, vomiting, diarrhea, chills, and fever

45. A 23-year-old G1 at 39 weeks’ gestation presents to triage with a chief complaint of


uterine contractions. They began 2 hours ago, are painful, and occur every 4 to 8
minutes. She reports good fetal movement, and no bleeding or leaking fluid. The
external tocometer shows contractions every 5 to 15 minutes. The fetal monitor shows
a category 1 tracing. On examination, her cervix is 1-cm dilated, 60% effaced, and the
fetal vertex is at -1 station. The patient had the same cervical examination in your
office last week. What is the most appropriate next step in management?
a. Send her home
b. Admit her for an epidural for pain control
c. Perform an amniotomy
d. Administer terbutaline
e. Augment her labor with Pitocin

The answer is a. This patient is most likely experiencing false labor, or Braxton-Hicks
contractions. False labor is characterized by contractions that are irregular in timing
and duration, and do not result in any cervical dilation. The intensity of Braxton-
Hicks contractions does not change, the discomfort is mainly felt in the lower
abdomen, and the pain is usually relieved by sedation. In the case of true labor, the
uterine contractions occur at regular intervals, tend to become increasingly more
intense over time, and results in progressive dilation and effacement of the cervix.
Sedation does not stop the discomfort. There are three stages of labor. The first stage
of labor is the interval between the onset of labor and full cervical dilation. The first
stage consists of a latent phase (with gradual cervical change), and an active phase
(characterized by rapid cervical change). The second stage of labor begins with
complete cervical dilation and ends with the delivery of the fetus. The third stage of
labor is the time from delivery of the fetus to expulsion of the placenta. Since this
patient is not in true labor, the best plan of management is to send her home and await
spontaneous labor.

46. A 19-year-old P0 at 41 weeks presents in spontaneous labor. Her membranes rupture


spontaneously after she is admitted to labor and delivery, demonstrating meconium-
stained amniotic fluid. What is the best management strategy for this patient and fetus
at the time of delivery?
a. No special measures need to be taken, and this infant can be managed per the
normal routine.
b. The obstetrician should suction the oropharynx and nasopharynx on the perineum
after delivery of the head but before the delivery of the shoulders (intrapartum
suctioning).
c. A pediatrician should be called to the delivery in order to perform intubation of the
neonate.
d. A pediatrician should be called to perform routine tracheal suctioning.
e. A pediatrician should be called, and if the newborn is depressed, they should
intubate the trachea and suction meconium or other aspirated material from beneath
the glottis.

The answer is e. In 2006, the American Academy of Pediatrics and the American
Heart Association published new guidelines on neonatal resuscitation, which the
American College of Obstetricians and Gynecologists adopted as well. Infants with
meconium-stained amniotic fluid should no longer receive routine intrapartum
suctioning, which used to be the standard of care. Current evidence no longer supports
this practice, because routine intrapartum suctioning has not been shown to prevent or
alter the course of meconium aspiration syndrome. If the newborn is depressed,
tracheal suctioning should be undertaken. Attempted intubation of a vigorous
newborn may potentially result in more injuries to the vocal cords.

47. A 38-year-old G3P2 at 40 weeks’ gestation presents to labor and delivery with gross
rupture of membranes occurring 1 hour prior to arrival. The patient is having
contraction every 3 to 4 minutes on the external tocometer, and each contraction lasts
60 seconds. The FHR tracing is 120 beats per minute with accelerations and no
decelerations. The patient has a history of rapid vaginal deliveries, and her largest
baby was 3200 g. On cervical examination she is 5-cm dilated and completely
effaced, with the vertex at -2 station. The estimated fetal weight is 3300 g. The patient
is in a lot of pain and requesting medication. Which of the following is the most
appropriate method of pain control for this patient?
a. Intramuscular meperidine
b. Pudendal block
c. Perineal block
d. Epidural analgesia
e. General anesthesia

The answer is d. The most appropriate modality for pain control in this patient is
administration of an epidural analgesia. An epidural block provides relief from the
pain of uterine contractions and delivery. It is accomplished by injecting a local
anesthetic agent into the epidural space at the level of the lumbar intervertebral space.
An indwelling catheter can be left in place to provide continuous infusion of an
anesthetic agent throughout labor and delivery via a volumetric pump. In this patient,
intramuscular narcotics such as meperidine or morphine would not be preferred
because these agents can cause respiratory depression in the newborn if delivery is
imminent. A pudendal block involves local infiltration of the pudendal nerve, which
provides anesthesia to the perineum for delivery but no pain relief for uterine
contractions. A local perineal block refers to infusing a local anesthetic to the area of
an episiotomy. The inhalation of anesthetic gases (general anesthesia) is reserved
primarily for situations involving emergent cesarean deliveries and difficult
deliveries. All anesthetic agents that depress the maternal CNS cross the placenta and
affect the fetus. In addition, a major complication of general anesthesia is maternal
aspiration, which can result in aspiration pneumonitis.

Questions 48 and 49

A 35-year-old G2P1 at 39 weeks’ gestation presents to labor and delivery in active


labor. Her cervix is 5-cm dilated and 80% effaced, and the vertex is at 0 station. The
tocometer shows that she is having contractions every 3 minutes. The fetal heart
tracing shows a baseline rate of 140 beats per minute, moderate variability, with
accelerations and no decelerations.

48. This FHR tracing may best be interpreted as which of the following?
a. Category I
b. Category II
c. Category III
d. Category IV
e. Category V

The answer is a. In 2008, a workshop sponsored by the American College of


Obstetricians and Gynecologists, the National Institute of Child Health and Human
Development, and the Society for Maternal-Fetal Medicine convened to update fetal
heart tracing nomenclature and interpretation. A normal baseline FHR is 110 beats per
minute to 160 beats per minute. Moderate (normal) variability is an amplitude range
of 6 beats per minute to 25 beats per minute. An acceleration is an abrupt increase in
the FHR, with onset to peak in less than 30 seconds. If an acceleration lasts 10
minutes or longer, it is a baseline change. The committee adopted a threetiered FHR
interpretation system. Category I tracings include: baseline FHR 110 beats per minute
to 160 beats per minute, moderate variability, no decelerations, and either presence or
absence of accelerations. A category II tracing includes all tracings not categorized as
category I or III. These tracings require evaluation, increased surveillance, initiation
of corrective measures where appropriate, and reevaluation. They may include
tachycardia, minimal variability, absence of induced accelerations after fetal
stimulation, or episodic decelerations. Category III tracings include either absent
variability with recurrent late decelerations, recurrent variable decelerations, or
bradycardia, or a sinusoidal pattern. Category III tracings are abnormal and indicate
an increased risk for fetal academia. If unresolved, these tracings usually require
prompt delivery.

49. One hour after she is admitted, her membranes rupture spontaneously. Shortly
thereafter, she develops recurrent variable decelerations. What is the best next step in
management?
a. Continue to monitor, as variable decelerations do not require intervention
b. Initiate an amnioinfusion
c. Since she is remote from delivery, perform a cesarean
d. Administer oxygen by nasal cannula
e. Administer terbutaline

The answer is b. Intermittent variable decelerations by definition occur with fewer


than 50% of contractions. They are the most common FHR abnormality that occurs
during labor, and typically do not require treatment. Recurrent variable decelerations
by definition occur with greater than or equal to 50% of contractions and may be
more indicative of impending fetal academia. Management of recurrent variables
should include relieving umbilical cord compression. Changing the maternal position
is a reasonable first step. Amnioinfusion has been shown to decrease both recurrent
variables and the cesarean delivery rate. Terbutaline and oxygen are not indicated for
treatment of recurrent variable decelerations.

50. A 29-year-old G2P1 at 40 weeks is in active labor. Her cervix is 5-cm dilated,
completely effaced, and the vertex is at 0 station. She is on oxytocin to augment her
labor, and she has just received an epidural for pain management. The nurse calls you
to the room because the FHR has been in the 70s for the past 3 minutes. The
contraction pattern is noted to be every 3 minutes, each lasting 60 seconds, with return
to normal tone in between contractions. The patient’s vital signs are: blood pressure
90/40 mm Hg, pulse 105 beats per minute, respiratory rate 18 breaths per minute, and
temperature 36.1°C (97.6°F). On repeat cervical examination, the vertex is well
applied to the cervix and the patient remains 5-cm dilated and at 0 station, and no
vaginal bleeding is noted. Which of the following is the most likely cause for the
deceleration?
a. Cord prolapse
b. Epidural analgesia
c. Pitocin
d. Placental abruption
e. Tachysystole

The answer is b. A prolonged FHR deceleration is a decrease in the FHR that is 15


beats per minute or more, lasting 2 minutes or longer, but less than 10 minutes from
onset to return to baseline. Epidural analgesia is a common cause of FHR
decelerations because it can be associated with maternal hypotension and decreased
placental perfusion. Therefore, maternal blood pressure should always be noted in
cases of FHR decelerations. If maternal blood pressure is abnormally low, ephedrine
can be given to correct the hypotension. Because an umbilical cord prolapse can be
associated with decelerations, the patient should undergo a cervical examination to
evaluate for a prolapsed cord. In addition, the Pitocin infusion should be discontinued
to reduce uterine contraction frequency. The patient should be turned to the left lateral
position to decrease uterine pressure on the great vessels and enhance uteroplacental
flow. Supplemental oxygen should be given to the patient in an attempt to increase
oxygen to the fetus. A cesarean delivery may be performed if the FHR does not
respond to these resuscitative measures.

Questions 51 and 52

You are delivering a 26-year-old G3P2002 at 40 weeks’ gestation. She has a history
of two previous uncomplicated vaginal deliveries, and has had no problems during
this pregnancy. After 15 minutes of pushing, the baby’s head delivers spontaneously,
but then retracts back against the perineum. As you apply gentle downward traction to
the head, the baby’s anterior shoulder fails to deliver.

51. Which of the following is the best next step in the management of this patient?
a. Call for help
b. Cut a symphysiotomy
c. Instruct the nurse to apply fundal pressure
d. Perform a Zavanelli maneuver
e. Push the baby’s head back into the pelvis

The answer is a. In this clinical scenario, a shoulder dystocia is encountered. A


shoulder dystocia occurs when the fetal shoulders fail to spontaneously deliver
secondary to impaction of the anterior shoulder against the pubic bone after delivery
of the head has occurred. Shoulder dystocia is an obstetric emergency; the first step
should always be to call for help when such a situation is encountered. An episiotomy
may be necessary to allow the obstetrician to have adequate room to perform a
number of manipulations to try to relieve the dystocia. Such maneuvers include the
following—suprapubic pressure, McRoberts maneuver (flexing maternal legs upon
the abdomen), Wood’s corkscrew maneuver (rotating the posterior shoulder), Rubin
maneuver (rotate accessible shoulder toward anterior surface of the chest), and
delivery of the posterior shoulder (sweeping the posterior arm across the chest). There
is no role for fundal pressure, because this action further impacts the shoulder against
the pubic bone and makes the situation worse. A Zavanelli maneuver is replacement
of the fetal head into the pelvis so that cesarean delivery can be performed. It should
only be attempted when all other methods have failed. A symphysiotomy involves
cutting the pubic symphysis and has a high morbidity for the mother.

52. After performing the appropriate maneuvers, the baby finally delivers. The
pediatricians note that the right arm is hanging limply to the baby’s side with the
forearm extended and internally rotated. Which of the following is the baby’s most
likely diagnosis?
a. Erb palsy
b. Klumpke paralysis
c. Humeral fracture
d. Clavicular fracture
e. Paralysis from intraventricular bleed

The answer is a. Shoulder dystocias can be associated with significant fetal morbidity
including brachial plexus injuries, clavicular fractures, and humeral fractures.
Fractures of the clavicle and humerus usually heal rapidly and do not have any long-
term orthopedic or neurologic consequences. Brachial plexus injury usually results
from downward traction on the brachial plexus during delivery of the anterior
shoulder. Injury to the brachial plexus may be localized to the upper or lower roots. In
Erb palsy, the upper roots of the brachial plexus are injured (C5-C6), resulting in
paralysis of the shoulder and arm muscles; the arm hangs limply to the side and is
extended and internally rotated. In the case of Klumpke paralysis, the lower nerves of
the brachial plexus are affected (C7-T1) and the hand is paralyzed.

53. A 41-year-old G1P0 at 39 weeks, who has been completely dilated and pushing for 4
hours, has an epidural in place and remains undelivered. She is exhausted and crying
and tells you that she can no longer push. Her temperature is 38.3°C (101°F). The
FHR is in the 190s with decreased variability. The patient’s membranes have been
ruptured for over 24 hours, and she has been receiving intravenous penicillin for a
history of colonization with group B streptococcus. The fetal head is in the direct OA
position and is visible at the introitus between pushes. Extensive caput is noted, but
the fetal bones are at the +3 station. Which of the following is the most appropriate
next step in the management of this patient?
a. Perform a cesarean delivery
b. Encourage the patient to continue to push after a short rest
c. Attempt operative vaginal delivery
d. Rebolus the patient’s epidural
e. Cut anepisiotomy

The answer is c. Indications for an operative vaginal delivery with a vacuum extractor
or forceps occur in situations where the fetal head is engaged, the cervix is completely
dilated, and there is a prolonged second stage. It may also be indicated when there is
suspicion of potential fetal compromise, or need to shorten the second stage for
maternal benefit. In this clinical scenario, all of these indications for operative
delivery apply. This patient has been pushing for 4 hours, which meets criteria for
protracted second stage of labor in a nulliparous patient with an epidural, based on
contemporary data. In addition, potential maternal and fetal compromise exists since
the patient has the clinical picture of chorioamnionitis, and the FHR is nonreassuring.
It is best to avoid cesarean delivery if possible, since it would take more time to
achieve, and because the patient is infected.

54. A 28-year-old G1 at 38 weeks had a normal progression of her labor. She has an
epidural and has been pushing for 2 hours. The fetal head is direct occiput anterior at
+3 station. The FHR tracing is 150 beats per minute with recurrent variable
decelerations. With the patient’s last push, the FHR had a prolonged deceleration to
the 80s for 3 minutes. You recommend operative vaginal delivery due to the
nonreassuring FHR tracing. Compared to the use of the vacuum extractor, forceps are
associated with an increased risk of which of the following neonatal complications?
a. Cephalohematoma
b. Retinal hemorrhage
c. Jaundice
d. Intracranial hemorrhage
e. Corneal abrasions

The answer is e. Corneal abrasions and ocular trauma are more common with forceps
versus the vacuum, unless the vacuum is inadvertently placed over the eye. Vacuum
deliveries have a higher rate of neonatal cephalohematomas, scalp lacerations, retinal
hemorrhages, intracranial hemorrhages, and jaundice.

55. You performed a forceps-assisted vaginal delivery on a 20-year-old G1 at 40 weeks


for maternal exhaustion. The patient had pushed for 3.5 hours with an epidural for
pain management. A second-degree episiotomy was cut to facilitate delivery. Eight
hours after delivery, you are called to see the patient because she is unable to void and
complains of severe pain. On examination you note a large fluctuant purple mass
inside the vagina. What is the best management for this patient?
a. Apply an ice pack to the perineum
b. Embolize the internal iliac artery
c. Incise and evacuate the hematoma
d. Perform dilation and curettage to remove retained placenta
e. Place a vaginal pack for 24 hours

The answer is c. The described mass is a vaginal hematoma. Following operative


vaginal delivery, the symptoms of severe pain and urinary retention should lead to a
vaginal examination to evaluate for a fluctuant mass consistent with hematoma. Small
vulvar hematomas identified postpartum may be treated expectantly. If severe pain
occurs, or if the hematoma continues to expand, the best treatment is incision and
evacuation of the blood clots, with ligation of the bleeding vessels if they can be
identified. Often no sites of active bleeding are found, in which case the defect should
be closed and the vagina packed for 12 to 24 hours. Laparotomy may be indicated if
the hematoma extends into the broad ligament. Embolization of the vaginal branch of
the internal pudendal artery, uterine artery, and internal pudendal artery can be
performed if bleeding is intractable.

56. A 20-year-old G1 at 41 weeks has been pushing for 2½ hours. The fetal head is at the
introitus and beginning to crown. It is necessary to cut an episiotomy. The tear
extends through the sphincter of the rectum, but the rectal mucosa is intact. How
should you classify this type of episiotomy?
a. First-degree
b. Second-degree
c. Third-degree
d. Fourth-degree
e. Mediolateral episiotomy

The answer is c. A first-degree tear involves the vaginal mucosa or perineal skin, but
not the underlying tissue. In a second-degree laceration, the underlying subcutaneous
tissue (the fascia and muscles of the perineal body) is also involved, but not the rectal
sphincter or rectal mucosa. Thirddegree lacerations involve the anal sphincter. A
fourth-degree laceration involves a tear that extends into the rectal mucosa to expose
the lumen of the rectum.

57. A 16-year-old G1P0 at 38 weeks’ gestation presents to labor and delivery for the
second time during the same weekend that you are on call. She initially presented at
2:00 pm Saturday afternoon complaining of regular uterine contractions. Her cervix
was 1-cm dilated, 50% effaced with the vertex at −1 station, and she was sent home
after walking for 2 hours in the hospital without any cervical change. It is now Sunday
night at 8:00 pm, and the patient returns to labor and delivery with increasing pain.
She is exhausted because she did not sleep the night before because her contractions
kept waking her up. The patient is placed on the external fetal monitor. Her
contractions are occurring every 2 to 3 minutes. You reexamine the patient and
determine that her cervix is unchanged. Which of the following is the best next step in
the management of this patient?
a. Perform artificial rupture of membranes to initiate labor
b. Administer an epidural
c. Administer Pitocin to augment labor
d. Achieve cervical ripening with prostaglandin gel
e. Administer 10 mg intramuscular morphine

The answer is e. This patient is either experiencing prolonged latent labor or is in false
labor. Friedman criteria for the normal progress in labor were established in the
1950s, and until recently, were used for assessment and management of labor.
Friedman data indicated that the transition from the latent to the active phase of labor
occurred around 4 cm of cervical dilation, and that active phase dilation should
proceed at 1.2 cm per hour for nulliparous women, and 1.5 cm per hour for
multiparous women. However, contemporary data suggests that changes in obstetric
and anesthesia practices have resulted in changes in typical labor, and therefore,
criteria for labor progress have been reviewed and revised based on data from the
Consortium for Safe Labor (CSL). This data showed that cervical change between 3
cm and 6 cm is much slower than originally thought, and that the active phase of labor
is more likely to start around 6 cm dilation. Women who dilate at a rate less than 1 cm
per hour before 6 cm are still likely to proceed to spontaneous vaginal delivery. Data
needed to establish the normal range for the duration of labor in the latent phase are
not readily available because the onset of the latent phase in most women occurs
outside the hospital and therefore cannot be accurately determined. One way to
manage a protracted latent labor is to administer a strong sedative such as morphine
along with intravenous fluids. This is sometimes referred to as “hydration and
sedation.” This is preferred over augmentation with Pitocin or performing an
amniotomy, because 10% of patients will actually have been in false labor. Patients
who are not truly in labor will usually stop contracting after administration of
morphine and hydration with rest. If a patient truly is in labor, then, after the sedative
wears off, she will have undergone cervical change and will have benefited from the
rest in terms of having additional energy to proceed with labor. An epidural would not
be recommended because the patient may not truly be in labor. There is no role for
cervical ripening in this patient, because if she is not in labor, she can go home and
wait for natural cervical ripening and spontaneous labor.

58. A 34-year-old G3P2 delivers a baby by spontaneous vaginal delivery. She had scant
prenatal care and no ultrasound, so she is anxious to know the sex of the baby. At first
glance you notice female genitalia, but on closer examination the genitalia are
ambiguous. Which of the following is the best next step in the evaluation of this
neonate?
a. Chromosomal analysis
b. Evaluation at 1 month of age
c. Pelvic ultrasound
d. Thorough physical examination
e. Laparotomy for gonadectomy

The answer is d. Ambiguous genitalia at birth is a medical emergency, not only for
psychological reasons for the parents, but also because hirsute female infants with
congenital adrenal hyperplasia (CAH) may die if undiagnosed. CAH is an
autosomally inherited disease of adrenal failure that causes hyponatremia and
hyperkalemia due to lack of mineralocorticoids. A thorough physical examination is
the best initial evaluation. While it will not provide the definitive diagnosis of the
gender, it can provide clues. Examination should include inspection and palpation of
the genitalia, palpation for gonads in the inguinal canal or labioscrotal folds,
evaluation for fused labia, evaluation for presence of a vagina or pouch, and
assessment for other nongenital dysmorphic features. The newborn should also be
quickly evaluated for presence of hyper- or hypotension, or signs of dehydration.
Karyotype, electrolyte analysis, blood or urine assays for progesterone, 17a-
hydroxyprogesterone, and serum androgens such as dehydroepiandrosterone sulfate
are essential to the workup as well. Pelvic ultrasound or MRI can detect ovaries or
undescended testes, but that is not the first step in management. Laparotomy or
laparoscopy is sometimes necessary for ectopic gonadectomy after puberty has
occurred.

59. A 24-year-old primigravid woman, who plans to breastfeed, decides to have a home
delivery. Immediately after the birth of a 4.1-kg (9-lb) newborn, the patient
experiences massive hemorrhage from extensive vaginal and cervical lacerations. She
is brought to the nearest hospital in shock. Over the course of 2 hours, nine units of
blood are transfused, and the patient’s blood pressure returns to a reasonable level. A
hemoglobin value the next day is 7.5 g/dL, and three more units of packed red blood
cells are given. The most likely late sequela to consider in this woman is which of the
following?
a. Hemochromatosis
b. Stein-Leventhal syndrome
c. Sheehan syndrome
d. Simmonds’ syndrome
e. Cushing syndrome

The answer is c. A disadvantage of home delivery is the lack of facilities to control


postpartum hemorrhage. The woman described in the question delivered a large baby,
suffered multiple soft tissue injuries, and went into shock, needing nine units of blood
by the time she reached the hospital. Sheehan syndrome seems a likely possibility in
this woman. This syndrome of anterior pituitary necrosis related to obstetric
hemorrhage can be diagnosed by 1-week postpartum, as lactation fails to commence
normally. Other symptoms of Sheehan syndrome include amenorrhea, atrophy of the
breasts, and loss of thyroid and adrenal function. The other presented choices for late
sequelae are less likely. Hemochromatosis would not be expected to occur in this
healthy young woman, especially since she did not receive prolonged transfusions.
Cushing, Simmonds, and Stein-Leventhal syndromes are not known to be related to
postpartum hemorrhage. It is important to note that home delivery is not a
predisposing factor to postpartum hemorrhage.

60. A 27-year-old G4P3 at 37 weeks presents to labor and delivery with heavy vaginal
bleeding and painful uterine contractions. A bedside ultrasound demonstrates a fundal
placenta. The patient’s vital signs are: blood pressure 140/92 mm Hg, pulse 118 beats
per minute, respiratory rate 20 breaths per minute, and temperature 37°C (98.6°F).
The fetal heart rate tracing reveals tachycardia with decreased variability and
intermittent late decelerations. She is taken to the OR for an emergency cesarean, and
delivers a male infant with Apgar scores of 4 and 9. When the placenta is delivered, a
large retroplacental clot is noted. The patient becomes hypotensive, and bleeding is
noted from the wound edges and her IV catheter sites. Which of the following blood
products will most quickly resolve her cause of hemorrhage?
a. Cryoprecipitate
b. Fresh frozen plasma (FFP)
c. Packed red blood cells
d. Platelets
e. Recombinant factor VII

The answer is b. This patient has a large placental abruption, which is the most
common cause of consumptive coagulopathy in pregnancy. The bleeding described
signifies that the patient has a significant coagulopathy with hypofibrinogenemia.
Prompt and vigorous transfusion is needed. Packed red blood cells will restore blood
volume and increase oxygen carrying capacity. FFP contains about 600 mg to 700 mg
of fibrinogen and will promote clotting, and is the best choice to quickly resolve her
cause of hemorrhage. Cryoprecipitate contains clotting factors and fibrinogen, but in a
much lower amount (200 mg) than FFP, and has no advantage over the use of FFP in
this bleeding patient. Recombinant factor VII can be used for the treatment of severe
obstetrical hemorrhage but will not be effective if fibrinogen is low. Platelet
transfusion is considered in bleeding patients with platelets less than 50,000.

Question 61 - 64
A 30-year-old G5P3 has undergone a repeat cesarean delivery. She wants to
breastfeed. Her past medical history is significant for hepatitis B infection,
hypothyroidism, depression, and breast reduction. She is receiving intravenous
antibiotics for endometritis. The baby latches on appropriately and begins to suckle.

61. In the mother, which of the following is a response to newborn suckling?


a. Decrease of oxytocin
b. Increase of prolactin-inhibiting factor (PIF)
c. Increase of hypothalamic dopamine
d. Increase of hypothalamic prolactin
e. Increase of luteinizing hormone—releasing factor

The answer is d. The normal sequence of events triggered by suckling is as follows:


through a response of the central nervous system, dopamine is decreased in the
hypothalamus. Dopamine suppression decreases production of PIF, which normally
travels through a portal system to the pituitary gland; because PIF production is
decreased, production of prolactin by the pituitary is increased. At this time, the
pituitary also releases oxytocin, which causes milk to be expressed from the alveoli
into the lactiferous ducts. Suckling suppresses the production of luteinizing hormone
—releasing factor and, as a result, acts as a mild (but not reliable) contraceptive
(because the midcycle surge of luteinizing hormone does not occur).

62. Which of the following aspects of her history might prevent this patient from
breastfeeding?
a. Maternal reduction mammoplasty with transplantation of the nipples
b. Maternal treatment with ampicillin
c. Maternal treatment with fluoxetine
d. Maternal treatment with levothyroxine
e. Past hepatitis B infection

The answer is a. There are very few contraindications to breastfeeding. Most


medications taken by the mother enter into breast milk to some degree. Breastfeeding
is inadvisable when the mother is being treated with antimitotic drugs, tetracyclines,
diagnostic or therapeutic radioactive substances, or lithium carbonate. Acute
puerperal mastitis may be managed quite successfully while the mother continues to
breastfeed. Reduction mammoplasty with autotransplantation of the nipple makes
breastfeeding impossible. However, there are reduction mammoplasty techniques that
do potentially allow for some amount of breastfeeding; this would be most likely in
patients who underwent a surgery where the areola and nipple were not completely
severed. Ampicillin or levothyroxine can be safely used by breastfeeding mothers. A
past history of hepatitis B is not a contraindication to breastfeeding. With some acute
viral infections such as hepatitis B, there is the possibility of transmitting the virus in
milk.

63. The patient asks you about the pros and cons of breastfeeding. Which of the following
is an accurate statement regarding breastfeeding?
a. Breastfeeding decreases the time to return of normal menstrual cycles.
b. Breastfeeding is associated with a decreased incidence of sudden infant death
syndrome.
c. Breastfeeding is a poor source of nutrients for required infant growth.
d. Breastfeeding is associated with an increased incidence of childhood obesity.
e. Breastfeeding is associated with a decreased incidence of childhood attention
deficit disorder.

The answer is b. According to the American Academy of Pediatrics, some of the


benefits of nursing include a decrease in infant diarrhea, urinary tract infections, ear
infections, and death from sudden infant death syndrome. Human milk is the ideal
food for neonates. It provides species- and age-specific nutrients for the baby. It has
immunological factors and antibacterial properties, and contains factors that act as
biological signals to promote cellular growth. Breastfeeding can delay the resumption
of ovulation and menses but should not be considered contraception.

64. The patient returns to see you in 6 weeks for a routine postpartum visit. She has been
nursing her baby without any major problems, and wants to continue to do so for at
least 9 months. She is ready to resume sexual activity and wants to know what her
options are for contraception. She is a nonsmoker, and her only other medication is a
prenatal vitamin. Which of the following methods may decrease her milk supply?
a. Intrauterine device (IUD)
b. Progestin only pill
c. Depo-Provera
d. Combination oral contraceptive
e. Condoms and spermicide

The answer is d. The use of an IUD, barrier methods, and hormonal contraceptive
agents containing only progestins are all appropriate methods of birth control for
breastfeeding women. It is best for nursing mothers to avoid estrogen-containing
contraceptives because estrogen preparations can inhibit lactation or decrease milk
supply.

65. On postpartum day 2 after a vaginal delivery, a 32-year-old G2P2 develops acute
shortness of breath and chest pain. Her vital signs are: blood pressure 120/80 mm Hg,
pulse 130 beats per minute, respiratory rate 32 breaths per minute, and temperature
37.6°C (99.8°F). She has new onset of cough. She appears to be in mild distress. Lung
examination reveals clear bases with no rales or rhonchi. The chest pain is
reproducible with deep inspiration. Cardiac examination reveals tachycardia with 2/6
systolic ejection murmur. Pulse oximetry shows an oxygen saturation of 88% on room
air, and oxygen supplementation is initiated. Which of the following is the best
diagnostic tool to confirm the diagnosis?
a. Arterial blood gas
b. Chest x-ray
c. CT angiography
d. Lower extremity Dopplers
e. Ventilation-perfusion scan

The answer is c. The patient most likely has a pulmonary embolism (PE). All three
components of Virchow’s triad are present during pregnancy and the postpartum
period: venous stasis, endothelial injury, and a hypercoaguable state. The reported
incidence of venous thromboembolism during pregnancy is 1 in 500 to 1 in 2000. PE
is the seventh leading cause of maternal mortality, responsible for 9% of maternal
deaths, and therefore, rapid diagnosis and treatment are critical. The classic triad—
hemoptysis, pleuritic chest pain, and dyspnea—appears in only 20% of cases. The
most common sign on physical examination is tachypnea (> 16 breaths/min).
Ventilationperfusion scans with large perfusion defects and ventilation mismatches
support the putative diagnosis of PE, but this finding can also be seen with atelectasis
or other disorders of lung aeration. Conversely, a normal ventilation-perfusion scan
suggests that massive PE is not the etiology of the clinical symptoms. To confirm the
diagnosis, a CT pulmonary angiography is the best diagnostic tool in this setting, and
has high sensitivity and specificity for the diagnosis of PE. An arterial blood gas will
confirm hypoxia, but not confirm PE as the cause. A chest x-ray could be done to rule
out other causes such as pulmonary edema or pneumonia, but will not make the
diagnosis of PE.

66. A 71-year-old G2P2 presents to your gynecology office for a routine examination.
She says she is very healthy and denies taking any medication. She has no history of
abnormal Pap smears and has only had one sexual partner in her lifetime. She is a
nonsmoker and has an occasional cocktail with her dinner. She does not have any
complaints. In addition, she denies any family history of cancer. The patient tells you
that she is a widow and lives alone in an apartment in town. Her grown children have
families of their own and live far away. She states that she is self-sufficient and
spends her time visiting friends and volunteering at a local museum. Her blood
pressure is 140/70 mm Hg. Her height is 5 ft 4 in and she weighs 130 lb. Which of the
following are the most appropriate screening tests to order for this patient?
a. Pap smear and mammogram
b. Pap smear, mammogram, and colonoscopy
c. Mammogram, colonoscopy, and bone densitometry
d. Mammogram, colonoscopy, bone densitometry, and TB skin test
e. Mammogram, colonoscopy, bone densitometry, TB skin test, and auditory testing

The answer is c. In postmenopausal women, routine screening for colon cancer is


recommended with a colonoscopy to be performed every 10 years. Alternatively,
flexible sigmoidoscopy can be performed every 5 years along with a yearly fecal
occult blood test. Mammography should be performed every 1 to 2 years in all
women 50 to 74 years of age. Postmenopausal women, who are not on hormone
replacement therapy, and all women 65 years or older should be screened for
osteoporosis with a DEXA scan to determine bone mineral density. Screening for
cervical cancer with Pap smears may be discontinued after the age of 65 years in
women with adequate negative prior screening results and no history of CIN II or
higher. Adequate prior screening is defined as three consecutive negative Pap smears
or two consecutive negative HPV co-test results within the previous 10 years, with the
most recent test performed in the last 5 years. Women with a history of CIN II, CIN
III, or adenocarcinoma in situ should continue screening for a total of 20 years after
treatment. Tuberculosis skin testing need to be performed only in individuals with
HIV infection, those who have close contact with individuals suspected of having TB,
who are IV drug users, who are residents of nursing homes or long-term-care
facilities, or who work in a profession that is health care related. This patient does not
have any risk factors that would necessitate TB testing. Auditory testing is not a
routine screening test.

67. A 72-year-old G5P5 presents to your office for well-woman examination. Her last
examination was 7 years ago, when she turned 65. She has routine checks and
laboratory tests with her internist each year. Her last mammogram was 6 months ago
and was normal. She takes a diuretic for hypertension. She is a retired school teacher.
Her physical examination is normal. Which of the following is the best vaccination to
recommend for this patient?
a. Diphtheria-pertussis
b. Hepatitis B vaccine
c. Influenza vaccine
d. Measles-mumps-rubella
e. Pneumocystis

The answer is c. Women older than 65 years should have all of the following
immunizations: tetanus-diphtheria booster every 10 years, influenza virus vaccine
annually, and a one-time pneumococcal vaccine. Hepatitis B vaccine would be
indicated only in individuals at high risk (ie, international travelers, intravenous drug
users, and their sexual contacts, those who have occupational exposure to blood or
blood products, persons with chronic liver or renal disease, or residents of institutions
for the developmentally disabled, and inmates of correctional institutions). Herpes
zoster is indicated for women older than 65 years if not previously immunized.

68. A 65-year-old G3P3 presents to your office for annual checkup. She had her last well-
woman examination 20 years ago when she had a hysterectomy for fibroids. She
reports no medical problems, except some occasional stiffness in her joints early in
the morning. She takes a multivitamin daily. Her family history is significant for
cardiac disease in both her parents and breast cancer in a maternal aunt at the age of
42 years. Her physical examination is normal. Which of the following is the most
appropriate set of laboratory tests to order for this patient?
a. Lipid profile and fasting blood sugar
b. Lipid profile, fasting blood sugar, and TSH
c. Lipid profile, fasting blood sugar, TSH, and CA-125
d. Lipid profile, fasting blood sugar, TSH, and urinalysis
e. Lipid profile, fasting blood sugar, TSH, urinalysis, and CA-125

The answer is d. Women older than 65 years should undergo cholesterol testing every
5 years, fasting glucose testing every 3 years, screening for thyroid disease with a
TSH every 5 years, and periodic urinalysis is recommended in women older than 65
years. CA-125 testing is not recommended for screening for ovarian cancer. There are
many benign conditions which can cause an elevated CA-125, such as pregnancy,
endometriosis, fibroids, menses, pelvic inflammatory disease, peritoneal disease, and
liver disease.

69. You are following up on the results of routine testing of a 68-year-old G4P3 for her
well-woman examination. Her physical examination was normal for a
postmenopausal woman. Her Pap smear revealed parabasal cells, her mammogram
and lipid profile was normal, and the urinalysis shows hematuria. Which of the
following is the most appropriate next step in the management of this patient?
a. Colposcopy
b. Endometrial biopsy
c. Renal sonogram
d. Urine culture
e. No further treatment or evaluation is necessary if the patient is asymptomatic.

The answer is d. A urinalysis that is positive for blood should be followed up with a
urine culture to evaluate for an asymptomatic urinary tract infection before further
workup is done or referral to a urologist is made. Parabasal cells on a Pap smear
indicate lack of estrogen, and are a normal finding in postmenopausal women. It
requires no further evaluation.

70. A 74-year-old woman presents to your office for well-woman examination. Her last
Pap smear and mammogram were 3 years ago. She has hypertension, high cholesterol,
and osteoarthritis. She stopped smoking 15 years ago, and does not use alcohol. Based
on this history, which of the following medical conditions should be this patient’s
biggest concern?
a. Alzheimer disease
b. Breast cancer
c. Cerebrovascular disease
d. Heart disease
e. Lung cancer

The answer is d. In order of decreasing incidence, the leading causes of death in


women older than 65 years are the following: diseases of the heart, cancer,
cerebrovascular diseases, chronic obstructive pulmonary diseases, Alzheimer disease,
diabetes, pneumonia and influenza, accidents, renal disease, and septicemia.
71. A 21-year-old G0 presents to your office for a routine annual gynecologic
examination. She reports that she has previously been sexually active, but currently is
not dating anyone. She has had three sexual partners in the past, and says she
diligently used condoms. She is a senior in college and is doing well academically and
has many friends. She lives at home with her parents and a younger sibling. She
reports her 80-year-old grandmother was recently diagnosed with breast cancer. She
has no other family history of cancer. She says she is healthy and has no history of
medical problems or surgeries. She smokes tobacco and drinks beer occasionally, but
denies any illicit drug use. Her menses started at the age of 13 years, and are regular
and light without dysmenorrhea. Her blood pressure is 90/60 mm Hg. Her height is 5
ft 6 in and she weighs 130 lb. Based on this patient’s history, what would be the most
likely cause of death if she were to die at the age of 21 years?
a. Suicide
b. Homicide
c. Motor vehicle accidents
d. Cancer
e. Heart disease

The answer is c. The leading causes of death in women between the ages of 20 and 24
years, in order of decreasing frequency, are as follows: injuries/accidents, suicide,
malignancy, homicide, heart disease, pregnancy complications, birth defects,
influenza and pneumonia, stroke, septicemia, and diabetes.

72. A 17-year-old G1P1 presents to your office for her yearly well-woman examination.
She had an uncomplicated vaginal delivery the previous year. She has been sexually
active for the past 4 years and has had four different sexual partners, but has been
monogamous in the previous year with the same partner. Her menses occurs every 28
days and lasts for 4 days. She denies any intermenstrual spotting, postcoital bleeding,
or vaginal discharge. She reports no tobacco, alcohol, or illicit drug use. Which of the
following are appropriate screening tests for this patient?
a. Gonorrhea and chlamydia screening
b. Gonorrhea, chlamydia, and cervical cancer screening
c. Gonorrhea, chlamydia, and syphilis screening
d. Gonorrhea, chlamydia, hepatitis B, hepatitis C, and syphilis screening
e. Gonorrhea, chlamydia, hepatitis B, hepatitis C, herpes simplex, and syphilis

The answer is a. Routine screening for sexually transmitted disease is not warranted
for all women; however, all sexually active women younger than 25 years of age
should be routinely screened for gonorrhea and chlamydia, and older women with risk
factors such as new or multiple partners, sex work, or concurrent STD should also be
screened. There is no routine screening recommended for hepatitis B virus or herpes
simplex virus. Hepatitis C screening should occur in those with risk factors such as
intravenous drug use, dialysis, partner with hepatitis C, multiple partners, and
received blood products prior to 1990. Syphilis screening should also occur in those
with risk factors such as sex work, confinement in an adult correction facility or men
having sex with men. Screening for cervical cancer should begin at the age of 21
years.

73. A 15-year-old woman presents to your office for her routine physical examination
while she is on summer break from school. She denies any medical problems or prior
surgeries. She had chicken pox at the age of 4 years. Her menses started at the age of
12 years and are regular. She has recently become sexually active with her 16-year-
old boyfriend. She states that they use condoms for contraception. Her physical
examination is normal. Which of the following vaccines is appropriate to administer
to this patient?
a. Hepatitis A vaccine
b. Human papilloma virus vaccine
c. Meningococcal vaccine
d. Pneumococcal vaccine
e. Varicella vaccine

The answer is b. It would be appropriate for this patient to receive a human papilloma
vaccination, since it is recommended for all previously unvaccinated women aged 9 to
26 years. She is not a candidate for the varicella vaccine since she has had chicken
pox. The hepatitis A vaccine is indicated for international travelers, illegal drug users,
and health care workers. The pneumococcal vaccine is indicated in
immunocompromised persons, those with chronic illnesses, and individuals older than
65 years. Meningococcal vaccination is recommended for college freshmen living in
dorms, asplenia, or travel or residence in countries where meningococcal disease is
endemic.

74. A 26-year-old woman presents to your office for her well-woman examination. She
reports no medical problems or prior surgeries. She states that her cycles are monthly.
She is sexually active and uses oral contraceptive pills for birth control. Her physical
examination is normal. She reports that her 43-year-old paternal aunt was recently
diagnosed with breast cancer and is undergoing treatment. She reports that her
paternal grandmother died from ovarian cancer at the age of 75 years. She wants
genetic testing (BRCA) for breast and ovarian cancer. Which of the following
statements regarding genetic testing for breast and ovarian cancer is true?
a. All female relatives of an individual with breast cancer should undergo genetic
testing.
b. Genetic testing detects all germline mutations associated with the BRCA1 and
BRCA2.
c. Genetic testing is only recommended for individuals with affected individuals on
the maternal side of the family.
d. Most cases of breast cancer are due to germline mutations in BRCA1 and BRCA2.
e. When possible, the genetic testing should begin with the person who has ovarian
cancer or early on-set breast cancer.

The answer is e. Germline mutations in BRCA1 and BRCA2 account for the vast
majority of families with hereditary breast and ovarian cancer syndrome.
Approximately 10% of cases of ovarian cancer and 3% to 5% of cases of breast
cancer are due to germline mutations in BRCA1 and BRCA2. In the general
population, it is estimated that approximately 1 in 300 to 1 in 800 individuals carry a
mutation in BRCA1 or BRCA2. For a woman with a BRCA1 mutation, the risk of
ovarian cancer is 39% to 46%. For a woman with a BRCA2 mutation, the risk of
ovarian cancer is 12% to 20%. The estimated lifetime risk of breast cancer with a
BRCA1 or BRCA2 mutation is 65% to 74%. Evaluating a patient’s risk for hereditary
breast and ovarian cancer syndrome should be a routine part of obstetric and
gynecologic practice. When evaluating a family history, it is important to remember
that breast cancer and ovarian cancer predisposing genes can be transmitted through
the father as well as the mother. If possible, genetic testing should begin with a person
in the family who has ovarian cancer or early onset breast cancer (affected
individual). For obstetrician–gynecologists, certain clinical criteria have been
developed to assist in determining which patients would benefit from a genetic risk
assessment. The first group of criteria includes those patients with greater than an
approximate 20% to 25% chance of having an inherited predisposition to breast
cancer and ovarian cancer and for whom genetic risk assessment is recommended.
The second group of criteria includes those patients with greater than an approximate
5% to 10% chance of having an inherited predisposition to breast and ovarian cancer
and for whom genetic risk assessment may be helpful. Although, in most cases, an
inherited predisposition to ovarian cancer is caused by mutations in BRCA1 or
BRCA2, current technology does not allow identification of all mutations that must
exist in these genes.

75. A 21-year-old woman presents with left lower quadrant pain. An anterior 7-cm firm
adnexal mass is palpated. Ultrasound confirms a complex left adnexal mass with solid
components that appears to contain a tooth. What percentage of these tumors is
bilateral? a. Less than 1
b. 2 to 3
c. 10
d. 50
e. Greater than 75

The answer is c. Benign cystic teratomas (dermoids) are the most common germ cell
tumors, and account for about 20% to 25% of all ovarian neoplasms. They occur
primarily during the reproductive years, but may also occur in postmenopausal
women and in children. Dermoids are usually unilateral, but 10% are bilateral.
Usually the tumors are asymptomatic, but they can cause severe pain if there is
torsion or if the sebaceous material perforates, spills, and creates a reactive peritonitis.

76. A 54-year-old woman is scheduled for laparotomy due to a pelvic mass. At the time
of exploratory laparotomy, a unilateral ovarian neoplasm is discovered that is
accompanied by a large omental metastasis. Frozen section diagnosis confirms
metastatic serous cystadenocarcinoma. Which of the following is the most appropriate
intraoperative course of action?
a. Excision of the omental metastasis and ovarian cystectomy
b. Omentectomy and ovarian cystectomy
c. Excision of the omental metastasis and unilateral oophorectomy
d. Omentectomy and bilateral salpingo-oophorectomy
e. Omentectomy, total abdominal hysterectomy, and bilateral salpingo-oophorectomy

The answer is e. The survival of women who have ovarian carcinoma varies inversely
with the amount of residual tumor left after the initial surgery. At the time of
laparotomy, a maximum effort should be made to determine the sites of tumor spread
and to excise all resectable tumor. Although the uterus and ovaries may appear
grossly normal, there is a relatively high incidence of occult metastases to these
organs; for this reason, they should be removed during the initial surgery. Ovarian
cancer metastasizes outside the peritoneum via the pelvic or para-aortic lymphatics,
and from there into the thorax and the remainder of the body. Therefore, a complete
staging procedure also includes pelvic washings, pelvic and abdominal exploration for
metastatic disease, appendectomy, and lymph node sampling.

77. A 68-year-old woman is seen for evaluation of a swelling in the right, posterior aspect
of her vaginal opening. She has noted pain in this area when walking and during
intercourse. At the time of pelvic examination, a mildly tender, firm mass is noted just
outside the introitus in the right vulva at approximately 8 o’clock. Which of the
following is the most appropriate treatment?
a. Marsupialization
b. Administration of antibiotics
c. Surgical excision
d. Incision and drainage
e. Observation

The answer is c. Although rare, adenocarcinoma of the Bartholin gland must be


excluded in women older than 40 years of age who present with a cystic or solid mass
in this area. The incidence peaks in women in their sixties. The appropriate treatment
in these cases is surgical excision of the Bartholin gland to allow for a careful
pathologic examination. In cases of abscess formation, both marsupialization of the
sac and incision with drainage as well as appropriate antibiotics are accepted modes
of therapy. In the case of the asymptomatic Bartholin cyst, no treatment is necessary.

78. A 51-year-old woman is diagnosed with invasive cervical carcinoma by cone biopsy.
Pelvic examination and rectal-vaginal examination reveal the parametrium to be free
of disease, but the upper portion of the vagina is involved with tumor. Intravenous
pyelography (IVP) and sigmoidoscopy are negative, but a computed tomography (CT)
scan of the abdomen and pelvis shows grossly enlarged pelvic and periaortic nodes.
This patient is classified at which of the following stages?
a. IIa
b. IIb
c. IIIa
d. IIIb
e. IV
The answer is a. Cervical cancer is still staged clinically, not surgically. Physical
examination, routine x-rays, barium enema, colposcopy, cystoscopy,
proctosigmoidoscopy, and IVP are used to stage the disease. CT scan results, while
clinically useful, are not used to stage the disease. The stage does not include
information about lymph node involvement. Stage I disease is limited to the cervix.
Stage Ia disease is preclinical (ie, microscopic), while stage Ib denotes macroscopic
disease that is clinical visible. Stage II invades beyond the uterus but not to the pelvic
side wall or lower third of the vagina. It may involve the upper vagina and/or the
parametrium. Stage IIa denotes tumor without parametrial invasion or involvement of
the lower third of the vagina, while stage IIb denotes parametrial extension. Stage III
involves the lower one-third of the vagina or extends to the pelvic side wall; there is
no cancer-free area between the tumor and the pelvic wall. Stage IIIa lesions have not
extended to the pelvic wall, but involve the lower one-third of the vagina. Stage IIIb
tumors have extension to the pelvic wall and/or are associated with hydronephrosis or
a nonfunctioning kidney caused by tumor. Stage IV is outside the reproductive tract,
such as invasion of the mucosa of the bladder or rectum.

Questions 79 to 82

A 45-year-old G1P1 presents for her routine annual examination. The patient is a
healthy smoker who has no medical problems. Her surgical history is significant for a
cesarean delivery with bilateral tubal interruption. You perform a Pap smear, which
returns showing high grade squamous intraepithelial lesion (HSIL). She undergoes
colposcopy, which is inadequate.

79. What is the next step in management?


a. Repeat Pap smear in 6 months.
b. HPV testing.
c. Cone biopsy.
d. Repeat Pap smear with co-testing in 1 year.
e. Repeat the colposcopy in 4 to 6 months.

The answer is c. An adequate colposcopy requires that the entire squamocolumnar


junction and all lesions be visualized, and that the biopsies of the lesion explain the
abnormal cytology. Since her colposcopy was not adequate, an excisional procedure
is required.

80. Cone biopsy of the cervix shows squamous cell cancer that has invaded only 2 mm
beyond the basement membrane with a lateral spread of 5 mm. There are no confluent
tongues of tumor, and there is no evidence of lymphatic or vascular invasion. The
margins of the cone biopsy specimen are free of disease. How should you stage this
patient’s disease?
a. Microinvasive cancer, stage Ia2
b. Carcinoma in situ
c. Microinvasive cancer, stage Ia1
d. Invasive cancer, stage Ib
e. Invasive cancer, stage IIa

The answer is c. Stage Ia1, or microinvasive carcinoma of the cervix, includes lesions
stromal invasion 3 mm or less in depth and 7 mm or less in horizontal spread, with no
confluent tongues or lymphatic or vascular invasion. Stage Ia2 is stromal invasion
more than 3 mm but less than 5 mm, and horizontal spread less than 7 mm.

81. Which lymph node group would be the first involved in metastatic spread of this
disease beyond the cervix and uterus?
a. Common iliac nodes
b. Sacral nodes
c. External iliac nodes
d. Paracervical nodes
e. Para-aortic nodes

The answer is d. The main routes of spread of cervical cancer include vaginal mucosa,
myometrium, paracervical lymphatics, and direct extension into the parametrium. The
prevalence of lymph node disease correlates with the stage of malignancy. Primary
node groups involved in the spread of cervical cancer include the paracervical,
parametrial, obturator, hypogastric, external iliac, and sacral nodes, essentially in that
order. Less commonly, there is involvement in the common iliac, inguinal, and
paraaortic nodes. The presence of lymph node involvement confers a worse prognosis
and impacts how the patient is managed. In stage I, the pelvic nodes are positive in
approximately 15% of cases and the para-aortic nodes in 6%. In stage II, pelvic nodes
are positive in 28% of cases and para-aortic nodes in 16%. In stage III, pelvic nodes
are positive in 47% of cases and para-aortic nodes in 28%.

82. This patient now asks you for your advice on how to treat her cervical cancer. Your
best recommendation is for the patient to undergo which of the following?
a. Treatment with external beam radiation
b. Implantation of radioactive cesium into the cervical canal
c. Extrafascial hysterectomy
d. Radical hysterectomy with pelvic lymphadenectomy
e. Treatment with adjuvant chemoradiation

The answer is c. The treatment of choice for microinvasive disease in a woman who
has completed childbearing is extrafascial (or simple) hysterectomy. If the patient
desired fertility sparing treatment, then stage Ia1 disease may be treated with cone
biopsy

Questions 83 and 84

83. A woman is found to have a unilateral invasive vulvar carcinoma that is 3 cm in


diameter but not associated with evidence of lymph node spread. Initial management
should consist of which of the following?
a. Chemotherapy
b. Radiation therapy
c. Simple vulvectomy
d. Radical vulvectomy with bilateral lymphadenectomy
e. Radical local excision and ipsilateral inguinal lymphadenectomy

The answer is e. Women who have invasive vulvar carcinoma usually are treated
surgically. Tumors larger than 2 cm are staged as IB. If the lesion is unilateral, is not
associated with fixed or ulcerated inguinal lymph nodes, and does not involve the
urethra, vagina, anus, or rectum, then treatment usually consists of radical excision
and unilateral inguinal lymphadenectomy. The risk of inguinal node metastasis is
around 8%. Inguinal lymphadenectomy involves removal of the superficial inguinal
and deep femoral lymph nodes. Unilateral rather than bilateral lymphadenectomy
decreases postoperative morbidity. The lymph nodes should be sent intraoperatively
for frozen section, and if positive, a bilateral lymphadenectomy should be performed.
Radiation therapy, though not a routine part of the management of women who have
early vulvar carcinoma, is employed in the treatment of women who have local,
advanced carcinoma.

84. If this woman had multiple medical comorbidities, what would be the best option for
management?
a. Chemotherapy.
b. Radiation therapy.
c. She should still undergo the same surgery that would be recommended for a healthy
patient.
d. Simple vulvectomy.
e. She should not receive any treatment, and should be referred to hospice.

The answer is b. Patients with multiple comorbidities who are not considered surgical
candidates should be treated with radiation therapy. In some institutions,
chemoradiation is preferred, but there is not good data to recommend that routinely

85. A pregnant 35-year-old patient is at highest risk for the concurrent development of
which of the following malignancies?
a. Cervix
b. Ovary
c. Breast
d. Vagina
e. Colon

The answer is c. Breast cancer is the most common type of malignancy detected
during pregnancy, affecting approximately 1 in 3,000 pregnant women. This is
thought to be at least partially due to the fact that more women are choosing to have
children later in life, and the risk of breast cancer increases with age.
86. A mother brings her 14-year-old daughter to the office for consultation. The mother is
concerned that her daughter is shorter than her friends, and should have started her
period by now. On physical examination, the girl is 4 ft 10 in tall. She shows evidence
of breast development at Tanner stage 2. She has no axillary or pubic hair. You
reassure the mother that her daughter seems to be developing normally. Educating the
mother and daughter, your best advice is to tell them which of the following?
a. The daughter will start her period when her breasts reach Tanner stage 5.
b. The daughter will start her period, then have her growth spurt.
c. The daughter’s period should start within 1 to 2 years since she has just started
developing breast buds.
d. The daughter will have her growth spurt, then pubic hair will develop, heralding the
onset of menstruation.
e. The daughter’s period should start by the age of 18 years, but if she has not had her
period by then, she should come back for further evaluation.

The answer is c. Significant emotional concerns develop when puberty is delayed. By


definition, if breast development has not begun by the age of 13 years, delayed
puberty should be suspected. Menarche usually follows about 1 to 2 years after the
beginning of breast development; if menarche is delayed beyond the age of 16 years,
delayed puberty should be investigated. Appropriate laboratory tests include
circulating pituitary and steroid hormone levels, karyotypic analysis, and CNS
imaging when indicated. An FSH value greater than 40 mIU/mL defines
hypergonadotropic hypogonadism as a cause of delayed pubertal maturation.
Hypergonadotropic hypogonadism is seen in girls with gonadal dysgenesis, such as
with Turner syndrome.

87. A mother brings her 12-year-old daughter to your office for consultation. She is
concerned because most of the other girls in her daughter’s class have already started
their period. She thinks her daughter hasn’t shown any evidence of going into puberty
yet. Knowing the usual first sign of the onset of puberty, you should ask the mother
which of the following questions?
a. Has your daughter had any acne?
b. Has your daughter started to develop breasts?
c. Does your daughter have any axillary or pubic hair?
d. Has your daughter started her growth spurt?
e. Has your daughter had any vaginal spotting?

The answer is b. In the United States, the appearance of breast buds (thelarche) is
usually the first sign of puberty, generally occurring between the ages of 9 and 11
years. This is subsequently followed by the appearance of pubic and axillary hair
(adrenarche or pubarche), the adolescent growth spurt, and finally menarche. On
average, the sequence of developmental changes requires a period of 4.5 years to
complete, with a range of 1.5 to 6 years. The average ages of adrenarche/pubarche
and menarche are 11.0 and 12.8 years, respectively; however, puberty onset is slightly
earlier in African-American girls and in overweight girls. These events are considered
to be delayed if thelarche has not occurred by the age of 13 years, adrenarche by the
age of 14, or menarche by the age of 16. Girls with delayed sexual development
should be fully evaluated for delayed puberty, including central, ovarian, systemic, or
constitutional causes.

88. A 9-year-old girl presents for evaluation of regular vaginal bleeding. History reveals
thelarche at the age of 7 and adrenarche at the age of 8. Which of the following is the
most common cause of this condition in girls?
a. Idiopathic
b. Gonadal tumors
c. McCune-Albright syndrome
d. Hypothyroidism
e. Tumors of the central nervous system (CNS)

The answer is a. In North America, pubertal changes before the age of 8 years in girls
and 9 years in boys are regarded as precocious. Although the most common type of
precocious puberty in girls is idiopathic, it is essential to ensure close long-term
follow-up of these patients to ascertain that there is no serious underlying pathology,
such as tumors of the CNS or ovary. Only 1% to 2% of patients with precocious
puberty have an estrogen-producing ovarian tumor as the causative factor. McCune-
Albright syndrome (polyostotic fibrous dysplasia) is also relatively rare and consists
of fibrous dysplasia and cystic degeneration of the long bones, sexual precocity, and
café au lait spots on the skin. Hypothyroidism is a cause of precocious puberty in
some children, making thyroid function tests mandatory in these cases. Tumors of the
CNS as a cause of precocious puberty occur more commonly in boys than in girls;
they are seen in about 11% of girls with precocious puberty.

Questions 89 to 91

A 68-year-old Caucasian woman comes to your office for advice regarding her risk
factors for developing osteoporosis. She is 5 ft 1 in tall and weighs 105 lb. She
stopped having periods at the age of 42 years. She is healthy and walks on a treadmill
daily. She does not take any medications. She has never taken hormone replacement
therapy (HRT). Her mother died at the age of 71 after she suffered a spontaneous hip
fracture.

89. Which of the following will have the least effect on this patient’s risk for developing
osteoporosis?
a. Her family history
b. Her race
c. Her level of physical activity
d. Her early menopause status
e. Her weight

The answer is e. Osteoporosis is defined as decreased bone mass and


microarchitectural disruption which leads to an increased risk of fractures. Fractures
may occur in the vertebra, distal forearm (Colles’ fracture), or femur head. Although
all races experience osteoporosis, white and Asian women lose bone earlier and at a
more rapid rate than black women. Thin women and those who smoke are at
increased risk for developing osteoporosis. Other risk factors include advanced age,
history of prior fracture, long-term steroid therapy, or excessive alcohol use. Physical
activity increases the mineral content of bone in postmenopausal women.

90. The patient asks how and if she should be tested for osteoporosis. What is the best
method to screen her for osteoporosis?
a. Peripheral measurement of her heel with photon absorptiometry
b. Standard x-ray of her spine
c. Dual-energy x-ray absorptiometry (DEXA)
d. Measure biochemical markers of bone remodeling
e. CT scan to measure the bone density.

The answer is c. DEXA is the best method to measure bone density because it is
precise, uses low doses of radiation, and has been well studied in terms of how the
DEXA results correlate to risk of fracture. In addition, the World Health Organization
(WHO) criteria for diagnosis for osteoporosis and osteopenia are based on DEXA
results. CT scan may measure bone density, but requires much higher doses of
radiation. Peripheral measurement may correlate to DEXA, but the results are difficult
to interpret. X-ray is not used to assess bone density.

91. The patient has a DEXA study that demonstrates a T-score of –2.0. What is the best
next step in management?
a. Begin a bisphosphonate
b. Encourage her to engage in weight bearing exercise and take a calcium supplement
c. Repeat the study in 1 year
d. Begin raloxifene therapy
e. Recommend she begin combined HRT

The answer is a. Osteopenia is defined as a T-score between –1 and –2.5 standard


deviations from the mean (SD). Osteoporosis is defined as a T-score below –2.5 SD.
A T-score is the standard deviation between the patient and the peak young adult bone
mass. The more negative, the greater the risk of fracture. This patient has osteopenia
based on her DEXA, and she also has risk factors for fracture (low body weight,
family history, of fracture), making her a good candidate for pharmacologic therapy to
reduce her risk of fracture. Bisphosphonates are considered first line therapy as they
are well tolerated, relatively low cost, and have a favorable safety profile. Raloxifene,
a selective estrogen receptor modulator (SERM), is effective at reducing the risk of
vertebral fractures, but is associated with an increased risk of thromboembolism;
therefore, it is usually used in situations where bisphosphonates are not well tolerated.
HRT is not indicated for the management of osteoporosis or osteopenia.

Questions 92 and 93
A 46-year-old P3003 presents to your office with a chief complaint of leakage of
urine. She reports that she leaks when she coughs or sneezes. She is otherwise
healthy, does not smoke, and takes no medications. Her history is significant for three
vaginal deliveries.

92. Which of the following is the most common cause of urinary incontinence in women
of this age?
a. Functional incontinence
b. Urge incontinence
c. Stress urinary incontinence (SUI)
d. Urethral diverticulum
e. Overflow incontinence

93. If this woman were 78-year-old, what would be the most likely cause of urinary
incontinence?
a. Anatomic stress urinary incontinence
b. Urethral diverticulum
c. Overflow incontinence
d. Urge incontinence
e. Fistula

The answers are 92-c, 93-d. SUI is the involuntary loss of urine when intravesical
pressure exceeds the maximum urethral pressure in the absence of detrusor activity. It
is often brought on by laughing, coughing, or sneezing. This incidence is highest in
women between the ages of 45 and 50. SUI may be caused by urethral hypermobility
or intrinsic sphincter deficiency (ISD). The other major cause of incontinence is urge
incontinence. With urge incontinence, the bladder leaks urine due to involuntary,
uninhibited detrusor contractions. The incidence of urge incontinence increases with
age. Other causes of urinary incontinence are less common and include overflow
incontinence secondary to urinary retention, congenital abnormalities, infections,
fistulas, and urethral diverticula. Urethral diverticula classically present with dribbling
incontinence after voiding. Functional incontinence occurs when a patient cannot
reach the toilet in time due to physical, cognitive, or psychological limitations. In the
elderly population there are also many transient causes of incontinence that the
physician should consider. These include dementia, medications (especially α-
adrenergic blockers), decreased patient mobility, endocrine abnormalities
(hypercalcemia, hypothyroidism), stool impaction, and UTIs.

94. A healthy 59-year-old woman with no history of urinary incontinence undergoes


vaginal hysterectomy with anterior and posterior (A&P) repair for uterine prolapse, a
large cystocele, and a rectocele. Two weeks postoperatively, she presents to your
office with a new complaint of intermittent leakage of urine. What is the most likely
cause of this complaint following her surgery?
a. Urethral diverticulum
b. Overflow incontinence
c. Rectovaginal fistula
d. Stress urinary incontinence
e. Vesicovaginal fistula

The answer is d. Many patients who have uterine prolapse or a large protuberant
cystocele will be continent because of urethral obstruction caused by the cystocele or
prolapse. In fact, at times these patients may need to reduce the prolapse in order to
void. Following surgical repair, if the urethrovesical junction is not properly elevated,
SUI may result. This incontinence may present within the first few days to weeks
following surgery. Typically, patients undergoing hysterectomy for prolapse will be
evaluated with urodynamic or other studies to help determine if they are likely to leak
once normal anatomy is restored following surgery. If they are shown to leak when
the cystocele is reduced, the physician may recommend a concomitant procedure to
support the mid urethra, such as a mid-urethral sling, to avoid the development of SUI
postoperatively. Rectovaginal fistula would present with passage of stool from vagina.
Vesicovaginal fistula would present with continuous leakage of urine from the vagina.
Detrusor instability would have been present prior to her surgery.

95. A 53-year-old postmenopausal woman, G3P3, presents for evaluation of new onset
urinary leakage for the past 6 weeks. Which of the following is the most appropriate
first step in this patient’s evaluation?
a. Urinalysis and culture test
b. Urethral pressure profiles
c. Intravenous pyelogram
d. Cystourethrogram
e. Urethrocystoscopy

The answer is a. When patients present with urinary incontinence, a urinalysis and
culture tests should be performed to evaluate for acute cystitis. In patients diagnosed
with a UTI, treatment should be initiated, and then the patient should be reevaluated.
It is not uncommon for symptoms of urinary leakage to resolve after appropriate
therapy. After obtaining the history and physical examination and evaluating a
urinalysis and urine culture, only a few clinical tests are necessary in the initial
evaluation of the incontinent patient. Most women with incontinence can begin
conservative treatment based on history and examination alone. However, further
conservative evaluation may include a PVR urine volume, cough stress test, and
urinary diary. A PVR is determined by bladder catheterization after the patient has
voided; when a large amount of urine remains after voiding, infection and
incontinence may result. A cough stress test is performed by filling the bladder with
fluid, asking the patient to cough or Valsalva, and directly visualizing the urethra to
see if there is leakage.

96. A 38-year-old woman G4P4 is undergoing evaluation for fecal incontinence. She has
no known medical problems. Which of the following is the most likely cause of this
patient’s condition?
a. Rectal prolapse
b. Diabetes
c. Obstetric trauma
d. Early onset dementia
e. Excessive caffeine intake

The answer is c. The most common cause of fecal incontinence is obstetric trauma
that causes direct damage to the anal sphincter or to the pudendal nerve. The rectal
sphincter can be completely lacerated, but as long as the patient retains a functional
puborectalis sling, a high degree of continence will be maintained. Anal sphincter
weakness may also be caused by other nontraumatic etiologies, such as spinal cord
injury. Other causes of fecal incontinence include conditions that decrease rectal
sensation, such as dementia, central nervous system (CNS) disease, diabetes, or
multiple sclerosis. Therapy for fecal incontinence includes bulk-forming and
antispasmodic agents, especially in those patients presenting with diarrhea. All
caffeinated beverages should be stopped. Biofeedback and electrical stimulation of
the rectal sphincter are other possible conservative treatments. Surgical repair of a
defect is indicated when conservative measures fail, when the defect is large, or when
symptoms warrant a more aggressive treatment approach.

97. You are discussing surgical options with the family of an elderly patient with
symptomatic pelvic organ prolapse. Le Fort colpocleisis may be more appropriate
than vaginal hysterectomy and A&P repair for patients in which of the following
circumstances?
a. The patient is debilitated and in a nursing home
b. The patient has had postmenopausal bleeding
c. The patient has had endometrial hyperplasia
d. The patient has had cervical dysplasia that requires colposcopic evaluation
e. The patient has a history of urinary incontinence

The answer is a. Partial colpocleisis by the Le Fort procedure is reasonable for elderly
patients who are not good candidates for surgery as treatment for uterine prolapse.
The technique is appropriate for women who have a uterus in situ, and involves
partial denudation of opposing surfaces of the vaginal mucosa followed by surgical
apposition, thereby resulting in partial obliteration of the vagina. Small strips of
vaginal epithelium are left laterally, to allow an outlet for drainage or bleeding.
Patients who are candidates for this procedure must have no evidence of cervical
dysplasia or endometrial hyperplasia, and must have an atrophic endometrium. This
type of obliterative procedure is ideal for women who cannot tolerate a more
extensive surgery and who no longer plan to have vaginal intercourse. A similar
obliterative procedure may be performed in women who have already undergone
hysterectomy, as treatment for vaginal prolapse. Urinary incontinence can be a side
effect of these procedures, so many surgeons perform a concomitant incontinence
procedure at the same time. An A&P repair essentially involves excision of redundant
mucosa along the A&P walls of the vagina, at the same time strengthening the vaginal
walls by suturing the lateral paravaginal fascia together in the midline
98. A 20-year-old G0 and her partner, a 20-year-old man, present for counseling for
sexual dysfunction. Prior to their relationship, neither had been sexually active. Both
report no medical problems. In medical experience, which type of male or female
sexual dysfunction has the lowest cure rate?
a. Premature ejaculation
b. Vaginismus
c. Primary impotence
d. Secondary impotence
e. Female orgasmic dysfunction

The answer is c. In a 5-year follow-up study of couples treated by Masters and


Johnson, the cure rates for vaginismus and premature ejaculation approached 100%.
Orgasmic dysfunction was corrected in 80% of women, and secondary impotence
(impotence despite a history of previous coital success) resolved in 70% of men.
Primary impotence (chronic and complete inability to maintain an erection sufficient
for intercourse) had the worst prognosis, with cure reported in only approximately
50% of cases. Other therapists report very similar statistics

99. A 28-year-old G3P3 presents to your office for contraceptive counseling. She has no
history of medical problems or sexually transmitted diseases. You counsel her on the
risks and benefits of all contraceptive methods. Which of the following is the most
common form of contraception used by reproductive-age women in the United States?
a. Pills
b. Condom
c. Diaphragm
d. Intrauterine device (IUD)
e. Sterilization

The answer is a. The most recent report from the CDC in 2010 demonstrated that the
pills remain the most commonly used form of contraception among reproductive-age
women in the United States with 28% of women choosing this method. Female
sterilization (27%) and condoms (16%) were the next most commonly used methods
of contraception used by women in the United States.

100. A 21-year-old woman presents to your office for her well-woman examination. She
has recently become sexually active and desires an effective contraceptive method.
She has no medical problems, but family history is significant for breast cancer in a
maternal aunt at the age of 42 years. She is worried about getting cancer from taking
birth control pills. You discuss with her the risks and benefits of contraceptive pills.
You tell her that which of the following neoplasms has been associated with the use
of oral contraceptives?
a. Breast cancer
b. Ovarian cancer
c. Endometrial cancer
d. Hepatic cancer
e. Hepatic adenoma
The answer is e. COC pills have been extensively studied to determine if there is an
increased risk of neoplasms or cancer with use of these medications. Epidemiologic
studies demonstrate that use of COCs actually decreases the risk of ovarian and
endometrial cancers. There have been no studies that clearly demonstrate an
association between the use of COCs and breast cancer. A slightly higher risk of
cervical cancer has been observed in some studies of users of oral contraceptives. The
risk of developing benign liver adenomas (which can cause life-threatening
hemorrhage if they rupture) is increased somewhat in users of oral contraceptives, but
the risk of hepatic carcinoma is not increased.

You might also like